⋮⋮⋮
No.
274310
>>274298Чи завжди 2+2=4?
Як поділити на нуль і не заглючити матрицю?
Яка найбільша таємниця математики?
⋮⋮⋮
No.
274312
Я десь считав про мультивсесвіт. І дивився в марвелі. Струнщики подейкують, що таких всесвітів порядка 10^500. І в кожному своя фізика. А математика у всьому мультиверсі універсальна і однакова?
⋮⋮⋮
No.
274314
OP
>>274310Якщо коротко: так. Якщо не коротко, то є три способа це "обійти":
1) щось накшталт "нехай знак 2 позначає тепер 1+1+1", тоді 2+2 = 6.
2) "2+2=4" можно інтерпретувати як "Т доводе 2+2=4", якщо Т протиречива то там можна й довести що 2+2!=4.
3) Якась ультраридакальна філософська позиція типу "Delta^0-анти-реялізм", з нього би виходило що 2+2=4 не є абсолютною істиною а залежить від погляду (хоча захисту таких позицій я не бачив).
Теорія границь, напівекзотичні алг.структури (полукільце [0,+inf], колеса, арифметика на RP^1, арифметика на CP^1), IEEE754 стандарт (прогери), дільники нуля в кільцях, інфенетиземалі в неархімедових впорядкованих кільцях (нестандартний аналіз). Але "чесне" ділення на 0 тільки в 2 і 3, в 1,4 і 5 ділення на 0 нема просто є деякі операції які "щось схоже на ділення на 0 нагадують". Але і у випадку 2 і 3 то більш "синтаксичний цукор" ніж "реальна необхідність" такі структури разглядати.
Найбільша таємниця: чому вона існує коли могла би не існувати. А якщо обрати один математичний проєкт, то програма Ленглендса загальноприйнято найбільший проєкт сучасної математики, хоча я більш маргінальні питання люблю, типу чи існує когерентна теоретико-множинна істина.
>>274312> Струнщики подейкують, що таких всесвітів порядка 10^500 А математика у всьому мультиверсі універсальна і однакова?Так це лейндскейп-проблема, або стільки або нескінченність, залежно від того якого фізика питаєш, і це тільки гетеротичних теорій струн. Так, математика в кожному однакова. Є розділи математики яки вивчають "математичні всесвіти" (теорія множин й теорія топосів), там математика може бути трошки різною в різних всесвітах, а може й не трішки.
⋮⋮⋮
No.
274315
>>274314>"математичні всесвіти" (теорія множин й теорія топосів), там математика може бути трошки різною в різних всесвітах, а може й не трішки.Тобто може бути багато мультивсесвітів? Бо якщо мультивсесів з його 10^500 всесвітів об'єднує універсальна математика. То інакша математика то вже інший мультивсесвіт?
⋮⋮⋮
No.
274316
OP
>>274315Це трохи про різне. Теорії струн це QFT (квантові теорії поля), тобто це математичні девайси щоб рахувати складні ефекти з мікрочастичками на дуже високих енергіях. Ці розрахунки потребують додаткового вибору, і, якщо ти не супер сильно розогнав свою шизу, в тебе має бути тільки не більш одного вибіра який порахує ці складні ефекти "правильно" тобто "так як вони насправді в нашому всесвіті", ці підрахунки теоретично можна перевірити в супер дорогих експериментах (накшталт "коллайдер розміром з галактику"). Це питання відповідності мат. моделі реяльному всесвіту.
Топоси і моделі теорій множин вони про "інтелектуальний математичний всесвіт", і ніякої процедури перевірки немає, ти не можеш поставити експеримент який перевіряє "чи континуум гіпотеза істина чи ні", це просто немає сенсу, бо це питання не про відповідність мат. моделі реяльному всесвіту, а про відповідність мат.моделі "математичному всесвіту". Тому думаю що відповідь "ні".
⋮⋮⋮
No.
274317
>>274316Чи правильно я розумію, що оті всесвіти про які кажуть струнщики, це множина усіх можливих всесвітів які може реалізувати наша математика?
А оті "математичні всесвіти", то вже інша множина мультивсесвітів, в кожному з яких струнщик буде бачити свою множину всесвітів, згідно дійсної для того мультивсесвіту математики?
⋮⋮⋮
No.
274319
>>2742981. Якщо в математиці існують нескінченності (різних обсягових порядків), ірраціональні та уявні числа, то звідки вони беруться, і чи можна цей спосіб отримання або виявлення відобразити в реальності за допомогою умовних відповідників у фізиці матеріальних речей?
2. Які існують приклади використання математичних операцій з графами у 4d або гіперболічному просторі?
3. Яке в тебе враження від різномаїття вузлів, пляшок Кляйна та Розшарування Гопфа?
4. Чи може сума тригранного кута бути більшою за 360°?
5. Яким чином можна описати динаміку логічних перетворень одної об'ємної фігури в іншу, або з плоскої на об'ємну?
⋮⋮⋮
No.
274333
OP
>>2743191) Я думаю то різні процедури. Уявні числа це трохи про "геніяльне інженерне рішення" коли за допомогою старих інструментів й технологій робиш технологію яка всім потрібна, типу як велисопед: здається два колеса і залізна палка між ними, але за рахунок внутрішньої логіки їх взаємодії роботи виходить вирішувати нові задачі. Ірраціональні числа це про процедуру поповнення, ідеялізації, затикання дирок, фізично: це напевно коли дирки та відсутність матерії в конструкції сприймаєш як частину конструкції (electron hole?). Нескінченість різних порядків це скоріш про "винахід в природничіх науках", тобто коли ти якусь внутрішню структуру досліджуєш опосередковано через експеримент.
2) Дія абсолютної групи галуа на дитячих малюнках (dessin denfant), вкладенних в CP^1, використовується щоб надати геометричну інтерпретацію абсолютної групи галуа. Двоїсна триангуляція на чоторьохвимірному просторі може бути задіяна щоб надати комбінаторно-симплектичну інтерпретацію двоїсності Пуанкаре на цьому просторі, але графи і 4д там трохи посередньо. Взагалі 4д і графи погано один з одним працюють. Більше сенсу має разглядати многовимірні узагальнення графів (симпліціяльні комплекси) якщо хочеться сформулювати многовимірне питання про "щось типу графів". Там операцій просто купа. Надстройка, eds_k-підрозбиття наприклад.
3) Узлів стількі ж скількі простих (арифметична топологія, М2КР-словник) враження сильне. Пляшка клейна трохи як артефакт виглядає, ні для чого не потрібна. Розшарувань Гопфа чотири, виключні структури завжди любив і люблю, також сильне.
4) В гіперболічних просторах може.
5) Гомотопія/ізотопія/регулярна гомотопія або ізотопія/сімейство просторів (морфізм X -> S з додатковими обмеженнями)/гомологічна еквівалентність/лінійна або алгебрична або деформаційна еквівалентність в залежності від того що саме хочеш описати і скількі фіч при описі зберегти.
Ну а що.
⋮⋮⋮
No.
274336
Чудово.
Можеш пояснити теорему Байеса на частковому випадку любителей полизати жирну пизду.
⋮⋮⋮
No.
274339
OP
>>274336Є гарний відос у 3blue1brown де теорема Байеса інтерпретується як "апдейт впевненості в чомусь", раджу подивитись його, мені сподобалось.
⋮⋮⋮
No.
274344
OP
>4) В гіперболічних просторах може.
До речі я зараз не впевнений вже, ідея була в тому що "річчі-кривизна" це "типу щось з (n-1)-кутами", і якщо вона сильно <<0 то кути можно зробити великими, але тепер щось подумав що хуйня якась. В будь якому разі можно сконструювати такий простір штучно. Накриття ступені 2 над C розгалужене над 0 наприклад є "окружність" з центром в 0 довжиною 4 pi і одиничного радіуса. Але на такі питання взагалі без точної формалізації завжди можна "так" відповісти якщо їх достатньо широко інтерпретувати.
⋮⋮⋮
No.
274386
>>274344Наскільки це коректно відображає те, про що ти написав?
Коли простір розшаровується навпіл та закруглюється всередину, то тригранний кут стає менше.
Коли простір розривається від центру, тригранний кут збільшується... Максимально це до нескінченності, якщо я правильно розумію.
Тобто продовження закруглення краєвих складок можливе, там буде щось на кшталт кому сферичної форми з великою кількістю складок простору, трохи лячно від 11-вимірного простору.
⋮⋮⋮
No.
274391
OP
>>274386Я про те що двовимірна версія задачі про "чи завжди сумма кутів тригранного <2pi" це "чи завжди маленьке коло радіусу r має довжину 2 pi r + o(r)", якщо в тебе є дволисне накриття розгалужене в нулі як на пік то маленьке коло (= повний оберт) біля нуля має довжину 4 pi r + o(r).
В локально евклідових просторах (=ріманових многовидах) сумма плоских кутів тригранного кута завжди < 2pi, тому що в них будь яка задача про кути з однієї точки буде мати той самий розвʼязок що і в звичайному евклідовому. Твоя перша схема виглядає як локально-евклідовий простір, якщо я проінтерпретував правильно.
⋮⋮⋮
No.
274404
>>274386Половина з цих просторів це абстракція. Але я простий хуй, шо я знаю.
⋮⋮⋮
No.
274416
>>274415Я це розумію, але для чого там
>морал>фандаментал>праймалЦе філософія вже та біологія. Чи це просто спроба структуризувати? Тоді ок.
⋮⋮⋮
No.
274423
>>274416Думаю що так, якщо правильно питання зрозумів.
⋮⋮⋮
No.
274445
>>274404Так, то просто інтерпретація малограмотних езотериків, не більше.
Мав на увазі М-теорію з 11-ма вимірами. В самій Теорії Струн є також згадка про віртуальні частинки токсони та 27-вимірний простір. Дехто зазначає, що вимірів може бути нескінченна кількість, як сторін у кулі, але то вже забагато, достатньо 11.
11 Dimensions Explained - Higher Dimensions Explained - All Dimensions Explained #dimensions
⋮⋮⋮
No.
274450
>>274440"Самоперехрещується" це гарна інтерпретація, про "роширюється з середини назовні" або "переходить в проекції іншого виміру", я не впевнений що гарна. Про "роширюється з середини назовні" я би сказав що це +- відповідає тому що Ric<0 але навіть в цьому разі якщо простір локально евклідовий то сума триграного кута < 2pi, про "приходить з проєкції вищого виміру" - це можна про кожен простір сказати і це не є властивістю яку може мати або не мати простір.
Але я більше хотів би підкреслити що відповідь на це питаннє дуже нестабільна відносно інтерпретації того що розуміти під "простіром" (які сами просторо-подібні обʼєкти ми разглядаємо в якості дозволених), і може бути "так" або "ні" в залежності від цього.
>>274445Відос не дуже. Канонічно їх 10, а саме простір-час R^4 домножається на комплексно-трьохвимірне компакте калабі-яу X (кк-компактифікація), це Х воно не має ніякого додаткового інтерпретаційного навантаження (типу "всі можливі поссібілітіс" або "всі можливі можливості всіх можливих можливостей", чи щось таке), це саме додаткові просторовочасові виміри, які ми не помічаємо в повсякденному житті через те що вони маленькі. Про 11d процедура складніше але інтерпретації типу "всі можливі поссібілітіс" є максимально не туди.
⋮⋮⋮
No.
274458
>>274445Ось це відео більш конкретне:
The 11 Dimensions EXPLAINED>>274450Просто уявляю звідки в плоскості беруться 360°—від центра умовного кола простору відходять лінії до точок на периметрі.
Якщо цих точок периметра умовного кола буде більше, то це лише буде означати про континуум кутового діапазону, як безкінечна кількість чисел між 0 та 1.
Якщо сама лінія периметра викривлюється, відповідно точки з плоскості двовимірної площини у тривимірному просторі отримують нові координати відносно центра, то ті лінії, що йшли з центра до точок периметра, можуть роздвоюватись та перетинатись, і це "very mind-boggling".
⋮⋮⋮
No.
274462
OP
>>274458>Якщо цих точок периметра умовного кола буде більше, то це лише буде означати про континуум кутового діапазону, як безкінечна кількість чисел між 0 та 1.Периметр маленького кола поділити на радіус це безрозмірна величина тому вона не залежить в яких одиницях ти її вимірюєш, а залежить тільки від внутрішньої геометрії простора. Тому якщо періметр маленького кола поділити на радіус є 4pi, то це дійсно щось каже про те що "точок в діапазоні більше ніж очікується", не про одиниці в яких вимірюєш.
>Якщо сама лінія периметра викривлюється, відповідно точки з плоскості двовимірної площини у тривимірному просторі отримують нові координати відносно центра, то ті лінії, що йшли з центра до точок периметра, можуть роздвоюватись та перетинатись, і це "very mind-boggling".Для маленьких кіл і не дуже довгих ліній не можуть, принайні якщо простір = рімановий многовид, тому що в ріманових многовидах на маленьких відстанях все виглядає звичайно, а щоб кути з однієї точки міряти великі відстані не потрібні.
⋮⋮⋮
No.
274465
OP
>>274458Так там середина відоса с тією самою ідеєю про те що 7д це "всі можливі можливі можливості во всіх паралельних всесвітах", що взагалі ні, я не можу цьому сенс надати. Це здається розповсюджений якийсь мем який гуляє по екосистемі мутних ютуб-каналів. Остання частина відоса +- ок, тобто це те як це пояснюють в підручниках (окрім частини про планк-скейл).
⋮⋮⋮
No.
274472
>>274465Знайшов доволі цікаве роз'яснення:
"11-й вимір називається Теорією всього і пов’язаний з теорією струн.
Перші три виміри легко зрозуміти, але цікавий спосіб думати про 3-вимірні об’єкти: уявити лінію з відгалуженням, а потім скласти цю лінію, щоб з’єднати її з початковою лінією, від якої вона відгалужується, тож тепер це тривимірний об’єкт у вигляді літери Р.
Це дуже допомагає при обговоренні вищих вимірів.
Згинання вимірів разом набагато легше зрозуміти, ніж намагатися уявити об’єкт у 10-D.
Четвертий вимір схожий на малювання 2-х з цих тривимірних об’єктів, представлених у різних часових рамках, з'єднаних між собою лініями.
5-й вимір бере до уваги всі різні гілки, які може зробити плин часу, кожна з яких закінчується різним результатом.
Шостий вимір схожий на те, що ми робили з тривимірним об’єктом спочатку, згинаючи лінію, створену в 5-му вимірі. Це легше уявити, якщо тривимірний об'єкт є просто точкою в просторі-часі. Тепер уявіть букву Y з тривимірними об’єктами на кінцях цієї форми. Лінії, що з'єднують їх,—це четвертий вимір, точка розгалуження—5-й. Тепер, якщо ви зігнете одну з ліній на Y і торкнетеся нею іншої лінії, ви отримаєте шостий вимір. Потім ви можете подорожувати в часі за допомогою цього методу.
Сьомий вимір—це коли ви уявляєте всі ці лінії та точки як одну точку в цьому багатовимірному просторі. Якщо ви намалюєте цю ущільнену просторово-часову точку, що представляє всі результати одного рішення, а потім намалюєте більше крапок, що представляють усі можливі результати, а потім з’єднаєте їх лініями, у вас буде 7-й вимір. Ці колекції називають нескінченними.
8-й вимір—це місце, де ви зараз уявляєте всі можливі результати іншої початкової умови, і ви з’єднуєте ці дві нескінченності разом. Ці інші нескінченності можуть мати кардинально різні результати.
9-й вимір згинає ці виміри разом, щоб транспортувати об’єкти до та з різних нескінченностей.
10-й вимір—це коли ви об'єднуєте буквально все в одну велику масу. Ви берете до уваги все, що може коли-небудь статися в будь-якому всесвіті з будь-якого початку в будь-який заданий період часу.
11-й вимір насправді не існує, тому що насправді більше нічого не потрібно брати до уваги, але теорія суперструн стверджує, що саме в ньому розташовані струни, які складають адрони (субатомні частинки), з яких складається наш Всесвіт. Струни нібито одновимірні і нібито замінюють кварки. Ці струни обертаються зі швидкістю світла, і те, як вони вібрують і обертаються, визначає, чи є вони «кварками вгору» або «вниз». Ми насправді мало знаємо про 11-й вимір. він може навіть не існувати, але М-теорія доводить, що він існує."
https://www.quora.com/What-is-the-11th-dimension
⋮⋮⋮
No.
274488
>>274475То виходить в науці дохуя шизоїдів що не можуть розділити фізичне та абстрактне? Піздосі.
⋮⋮⋮
No.
274491
OP
>>274488Хто зна, я не фізик.
⋮⋮⋮
No.
274497
OP
>>274488Але я думаю що це розрізнення по-перше не важливо, а по-друге трохи складніше ніж ти намагаєшся показати. Ну наприклад, атом то реальність чи вже абстракція?
⋮⋮⋮
No.
274542
>>274497Реальність, їх можна побачити та візуалізувати за допомогою супір пупір мікроскопів. Це як бактерії.
А намагатись пристосовувати філософію до математики це як на мене крок до шизи. Не хочу discourage-ити наукове вільнодумство, але ж реально не налазить.
⋮⋮⋮
No.
274543
OP
>>274542Але ж мікроскопи електронні, не оптичні, тобто ця "візуалізація" то є по суті числовий графік роботи доволі хитрого прилада, аніж безпосередньо те що можна побачити оком (бо побачити оком в оптичний мікроскоп не можна принципово). Але добре, що скажеш про електрон?
Точно не впевнений про що ти: хто пристосовує філософію до математики і що саме не налазить на що?
⋮⋮⋮
No.
274544
>>274543В курсі про те. З такої самої точки зору можна не вірити у візуалізації планет та галактик отриманих неоптичним методом. Лазерно-шмазерно-супір-хвильові телескопи? Фуйня! А шо про електрон?
Ота пікча не налазить (моральний вимір???). Знову ж, я не вчений.
⋮⋮⋮
No.
274545
OP
>>274544Це так, але я ж про то й казав, що та пікча і той текст до науки не має відношення.
>А шо про електрон?Ну там зображення яке взагалі можна получити воно ще дальше від того що хочеться називати "фотографією електрона".
⋮⋮⋮
No.
274750
OP
підійму бо я пʼяний
⋮⋮⋮
No.
274765
Завжди цікавило одне питання...
За допомогою векторного добутку двох R3 векторів можна знайти вектор, який буде перпендикулярний до обох.
Взагалі він матиме ще ряд властивостей, таких як поворот проти годинникової стрілки, площа, кут, ось це все.
Так от якщо говорити конкретно про перпендикулярність, то в R2 можна мати схожу операцію для одного вектору: (-y, x).
І вона теж матиме властивість "повороту" в "правильному" напрямку проти годинникової стрілки.
Виходить так, що для випадків R3 і R2, операції скалярного добутку і (-y, x) можуть "добудувати" базис.
Тобто ми маємо N-1 векторів (2 для 3-вимірного випадку і 1 для 2-вимірного), а ці операції знаходять третій незалежний вектор, який ще й "повернений" в правильному напрямку.
Чи можна цю операцію "знаходження додаткового виміру" розширити на довільну кількість просторів?
Чи можемо ми маючи 3 вектори 4-вимірного простору знайти четвертий, який перпендикулярний до решти і "повернутий" в "правильний" бік?
Я часто бачив вивід формули векторного добутку через базисні вектори i-j-k, але такий вивід завжди здавався мені не дуже природним.
Кілька разів намагався вивести формулу сам через умови перпендикулярності (типу a•x=0, b•x=0, знаходимо x), але завжди заходив в глухий кут на цьому "повороті" проти годинникової стрілки.
Виходить просто "пряма" яка навіть не прив'язана до якихось одиниць довжини, ще й до того ж напрямок можна обрати довільно: вверх або вниз.
Якщо зрозуміло про що я говорю, чи є якийсь спосіб це обійти? Як через просту формулу вказати, що x повинен виходити такий, щоб a повертався в b проти годинникової стрілки?
Дякую.
⋮⋮⋮
No.
274812
OP
>>274765Так, то є просто зірка годжа від wedge-добутку n-1 векторів, тобто якщо в тебе є n-1 векторів v1,…,vn-1 то вектор який ти хочеш побудувати то є v_n = \star (v_1 /\ ... /\ v_n-1), див.
https://en.wikipedia.org/wiki/Hodge_star_operator Інакше кажучі, i-а координата вектора v_n то має бути v_n^i = (-1)^(n-i) det( (v1,…,vn-1)_i ) де (v1,…,vn-1)_i це (n-1)x(n-1) матриця з векторами v1,…,vn-1 по стовпчикам і видаленною строчкою i.
Наприклад, якщо n=2 та v1=(x,y) то формули дадуть v_2^1 = (-1)^(2-1) det(y) = -y, v_2^2 = (-1)^(2-2) det(x) = x, тобто вийде той самий (-y,x).
Вивести це можно з наступного: ти “хочеш” щоб для будь яких векторів w1,…,wn-1 виконувалась тотожність
det(w1,…,wn-1,vn) = “орієнтована площина паралелепіпеда натягнутого на v1,…,vn-1 і спроєектованого на гіперплощину породжену w1,…,wn-1”
інакше кажучі
det(w1,…,wn-1,vn) = det( (wi,vj) )
бо, наприклад для n=3 і будь яких w1,w2,v1,v2 виконується
det(w1,w2, v1 x v2) = det((wi,vj)) = (w1,v1)(w2,v2) - (w1,v2)(w2,v1) це тотожність біне-коши чи щось таке, я це називаю “визначення скалярного добутка на зовнішній степені простора”.
ну так ось, подставляючи e1,…,ei-1,ei+1,…,en в якості w1,…,wn-1 можно знайти i-у координату v_n, що я і зробив.
⋮⋮⋮
No.
274897
>>274812Займаєшся репетиторством?
⋮⋮⋮
No.
274898
OP
>>274897Ні, не дуже люблю шкільну математику пояснювати, а на більш серйозну репититори здебільшего не потрібні.
⋮⋮⋮
No.
274904
>>274812Формули, зірки і визначники це, звичайно, добре.
Але чи немає якогось покрокового виведення формули?
Я можу вивести формулу для відстані, кута між векторами, визначника і навіть оберненої матриці з зовсім-зовсім базової алгебри і геометрії.
Невже "перпендикуляр за правилом правої руки" вимагає на порядок складнішого виведення?
⋮⋮⋮
No.
274906
>>274898Роби ютуб канал.
Будеш пояснювати ОМРЕДУ за матан.
⋮⋮⋮
No.
274926
OP
>>274904Так про зірки то було для великої картини більше, мій вивід не опирався на них. Ну добре.
Перший крок це зрозуміти що ти хочеш порахувати. А саме по (л.н.) векторам v1,...,vn-1 ти хочеш порахувати вектор vn такий що
1) det(v1,...,vn)>0 "правило правої руки" (бо що інакше значить "правило правой руки" для довільного R^n?)
2) (vi,vn)=0, 1<=i<n
3) |vn|^2 = vol(v1,...,vn-1)^2
Друге, доведемо наступну теорему: для довільних векторів w1,...,wn-1 має місце тотожність
det(w1,...,wn-1,vn) = det( (wi,vj) ) (права частина 1<=i,j<=n-1)
Третє, давай подставимо e1,…,ei-1,ei+1,…,en (базисні вектори) замість w1,…,wn-1 в попередню формулу. Получимо
(-1)^(n-i) v_n^i = det( (v1,…,vn-1) з видаленою строчкою з номером i)
звідки
v_n^i = (-1)^(n-1) det( (v1,…,vn-1) з видаленою строчкою з номером i)
Якщо будуть додаткові питання (про те як довести теорему наприклад) то питай.
>>274906Та може колись і зроблю.
⋮⋮⋮
No.
274929
OP
Записав доведення.
Друге, доведемо наступну теорему: для довільних векторів w1,...,wn-1 має місце тотожність
det(w1,...,wn-1,vn) = det( (wi,vj) ) (права частина 1<=i,j<=n-1)
обидві частини рівності антисиметричні й полилінійні функції відносно w1,…,wn-1, тому разбиваючи кожний вектор wi по базісу v1,…,vn достатньо довести
det(v1,…,vn-1,vn) = det( (vi,vj))
права частина то є майже визначник визначник грама (розбитий по стовпчику), тобто
det(v1,…,vn-1,vn) = Gram(v1,….,vn) /|vn|^2
det(v1,…,vn-1,vn) = det(v1,….,vn)^2 /|vn|^2
|vn|^2 = det(v1,…,vn)
vol(v1,...,vn-1) |vn| = det(v1,…,vn)
остання рівність то “обʼєм є основа на висоту”, тому вона вірна і все доведено.
⋮⋮⋮
No.
275029
>>274926>>274929По-перше, мені треба час щоб все це нормально опрацювати...
По-друге, умови 1 і 3 виглядають так, наче ми намагаємось знайти щось, що вже знаємо.
Але мені здається визначник це саме те, чого мені бракує для задовільного розуміння.
Дякую.
⋮⋮⋮
No.
275031
>>275029Будь ласка, питай якщо щось незрозуміло.
Умови 1,2,3 це мені здається природнім узагальненням векторного добутку, а саме векторний добуток v x w це вектор ортогональний v і w (умова 2), довжина которого чисельно дорівнює площині паралелепипеду, натягнутому на v і w (умова 3) і орієнтованим за правилом правої руки (умова 1). Принайні це те геометричне визначення яке мені давали в школі.
Напевно, без визначников складно, але ти казав що формулу зворотньої матриці виписав, там же визначник в знаменнику, я думав що ти більш-меньш знаєш що воно таке.
⋮⋮⋮
No.
275042
>>275031Я виводив визначник чисто алгебраїчно.
Типу ax + by = u і cx + dy = v. З них пробуємо вивести x і y, а вже в них виникає дуже цікавий спільний дільник ad-cb. І так для будь якої кількості вимірів.
Про те що він пов'язаний з площею а його знак вказує на те як вектори повернуті один відносно одного я взагалі дізнався набагато пізніше.
⋮⋮⋮
No.
276808
>>274312>>274315Макс Тегмарк виділяє 4 рівні в ієрархії мультиверса, кожен наступний включає в себе попередній:
1 рівень — просто різні об'єми Хаббла (в котрих конфігурації матерії можут повторюватись) в рамках одного всесвіту. За даними WMAP наш всесвіт нескінченний і містить нескінченну кількість матерії, на високому рівні однорідну. До цих місць можна добратись, просто вони далеко від нас.
2 рівень — суп із бульбашок в океані вічної хаотичної інфляції Андрія Лінде, інфляція зупиняється в деяких місцях, утворюючи наш дефолтний простір-час. Лінде говорить, що в одній із таких бульбашок живемо і ми, а інфляція експоненційно продовжується поза межами нашого всесвіту і віддаляє нас від інших
3 рівень — квантово-механічний мультиверс багатосвітової інтерпретації квантової механіки. Ми точно не знаємо, чи скінченна чи нескінченна кількість цих всесвітів, і не знаємо як часто бранчиться хвильова функція. В людському тілі десь 5000 атомів проходять радіоактивний розпад щосекунди, якщо кожен розпад бранчить хвильову функцію на 2, то це 2^5000 нових всесвітів кожної секунди. Багацько.
4 рівень — це спекулятивний тегмарковський/ігановський математичний мультивсесвіт, де наш блок простору-часу
буквально являється статичною математичною структурою, але існують і інші когерентні складні і прості всесвіти, цей рівень також досліджується в романі
https://en.wikipedia.org/wiki/Permutation_City4 рівень стверджує, що немає різниці між фізичним існуванням та математичним. (що трохи нагадує модальний реалізм Льюїса
https://en.wikipedia.org/wiki/Modal_realism)
Тому щоб відповісти на твоє питання - так, математика всюди однакова, допоки ми знаходимось в межах однієї структури четвертого рівня. (цей рівень включає в себе всі попередні)
Наприклад, правильних многогранників завжди лише п'ять, ні більше, ні менше. По суті, вся математика об'єктивно існує, в своєму математичному визначенні слова "існування": із кожного набору аксіом слідує свій набір теорем, які вже "існують", і нам тільки залишається їх знайти. В інших математичних всесвітах аксіоматика може бути іншою, але, чесно кажучи, важко уявити як це.
Сурси:
https://arxiv.org/pdf/astro-ph/0302131.pdfhttps://arxiv.org/pdf/0704.0646/(мимо проходив, я не математик-кум)
⋮⋮⋮
No.
276814
Чи є математична модель підара? В мене виникла проблема теоретичної перевірки підарів, і я хочу системно підійти до цього питання. Рятуйте, безоси
⋮⋮⋮
No.
276872
>>276808Дякую за підйом нитки!
Прочитав Тегмарка, гарне, хоча 4 рівень мені не сподобався, у нього якесь занадто синтаксичне уявленя про те що таке "математична структура", постійно апелює до рівнянь, описів, алгоритмічної складності і таке, ну він фізик йому можна. Якщо розуміти "математична структура" більш
адекватно і розрізняти структуру і її опис то, по-перше, аргумент "more the rule than the exception — mathematical structures tend to have them by default" не працює, а по-друге, воно набагато меньш зрозуміло що то таке ніж він намагається це показати.
Ну і хвильова функція у системи завжди одна, бранчиться "світ з точкі зору frog перспективи" (в його термінології).
Інши системи аксиом для основ математики напівактивно вивчаються, так що це уявити нескладно. Вивчаються навіть питання по типу "як математику А бачить математика В" щоб трохи відійти від діхотомії "теорія/метатеорія" як від наївної.
⋮⋮⋮
No.
276873
>>276872А, він в другій статті каже про те що математична структура то є опис по модулю еквівалентності описів, так що добре, хоча дивно чому з його точкі зору не може існувати структури без опису.
⋮⋮⋮
No.
276968
>>276873Прочитав другий текст уважніше. Там він пише з більшою кількістю деталей про те що "математична структура" то є, і виявляється що він розуміє різницю між описом і самою структурою. Але все одно якось занадто мало уваги приділяє "загальним математичним структурам" і питанням про що воно таке є, тільки в секції VII-D.
До речі коли я казав про "різні математичні всесвіти" (в сенсі моделі теорії множин або елементарні топоси), я мав на увазі дещо що можна було би назвати "мультіверсом рівня 5", тобто набіром "різних математик" в середені якіх "однакові" математичні структури виглядають дещо по-різному. Він пише що рівня 5 не існує бо це теж можна назвати рівнем 4 бо це математична структура, але мені здається це трохи не так через ефекти схожі на парадокс-брехуна, які неможливо елімінувати (в інтерпретації Ніка Вівера). Про те чи існує "мультіверс рівня 5" чи то один юніверс це питання теорії множин останніх років і йдуть дебати (мультіверс Хамкінса й програма Ultimate-L Вудіна наприклад, але є й інші східні).
Але веселий текст, мені сподобався, добавив в закладинки.
⋮⋮⋮
No.
276969
> тобто набіром "різних математик" в середені якіх "однакові" математичні структури виглядають дещо по-різному.
Ну й сказав так сказав, я мав на увазі що кожна модель теорії множин або елементарний топос вона окремий мультівсесвіт рівня 4.
⋮⋮⋮
No.
277019
>>274298Чим займаєшся по життю? Який навчальний і кар'єрний план у тебе був що ти до цього докотився?
⋮⋮⋮
No.
277021
>>277019Він тобі вже не відповість. На кропивачі закрили доступ до сайту з іноземних айпі адрес.
⋮⋮⋮
No.
277033
>>277019Я аспірант на заході, карʼєрний план получити phd пройти 2-3 постдока а потім шукати тимчасову позицію професора (tenure track), а потім постійну. Не знаю чи вистачить в мене мотивації на таке бо це план ще років на 10. Є ще деякі ідеї по розвитку математики в Україні, але вони занадто розмиті щоб про них серйозно казати. Навчального плану нема, дивитись на те що корисно було би вивчити і вивчати це. Докотився природнім шляхом: якщо хочеш щоб за чисту математику давали грощі (я хочу) то ти залишаєшся в академії, особливо нема опцій інших.
>>277021В мене норд впн.
⋮⋮⋮
No.
277035
>>277021Пиздуй в інт, проксідебіл.
⋮⋮⋮
No.
277043
>>277033>Не знаю чи вистачить в мене мотивації на таке бо це план ще років на 10. Є ще деякі ідеї по розвитку математики в Україні, але вони занадто розмиті щоб про них серйозно казати. Я не математик (я той анон що вважає інтерпретацію неточних процесів типк філософії математикою першим кроком до шизи) але тбх відповідь на це є лише одна – брати дохуя грошей в Європи і хуячити гіперпотужну безкоштовну освіту і мотивувати навчальні заклади робити монетизуємий research замість бюджетування.
⋮⋮⋮
No.
277048
>>277043З моєї точки зору більше однієї. Достатньо декількох, порядка пʼяти, компетентних українськіх вчених у яких є бажання робити реальну математичну освіту в Україні за нічого, щоб навколо них згуртувались розумні першокурсники якіх засмучує застаріла програма і радянські методи навчання. В мене вже є стільки знайомих, але вони розгуртовані по світу, їх можна запрошувати читати курси на пів-року рік, а ще можна щось по інтернету вигадувати. Якщо коротко, то з моєї точкі зору потрібно одне маленьке але дуже потужне місце, на це не треба багато грошей. Але то все таке, поки нема розуміння як це могло би працювати.
Це про освіту, про науку, якщо коротко, в інматі не дуже чесно розподілення (існуючих) держ.грантів, їх получають тупі радянські діди і їх учні. Умовно, якщо би якийсь супер талановитий вчений, типу Вʼязовськой, чогось приїхав би в Україну і хотів би робити там математику, то взагалі не факт що він би зміг вибити держ.грант. Але як це змінити є ще меньш розуміння в мене.
> я той анон що вважає інтерпретацію неточних процесів типк філософії математикою першим кроком до шизиТочно не зроз про що ти, якщо про ті відяо з додатковими розмірностями простору, то з моєї точкі зору вони погані. Повторив щоб впевнений бути що один одного зрозуміли.
⋮⋮⋮
No.
277050
>>277049Я не думаю що треба переконувати людей з позицією "ано мне в жизні не панадабіца", по-перше якщо в них така позиція з самого початку то вони не будуть ту книжку читати, по-друге вони частково мають рацію, більшості людей шкільна математика після 9го класу в жизні буде не потрібна, по-третє в мене всеж такі більше надій на стратегію "зробити дуже потужну освіту але з таргет на малу кількість людей (групу з 30 першокурсників наприклад)", аніж на "переконати пів-країни що математика їм в житті буде потрібна через наукпоп".
Але я не один раз бачив попит на книжку "математика для дорослих які погано вчились в школі українською", що мене здивувало дуже, так що таку можливо й напишу (колись), є ідеї.
⋮⋮⋮
No.
277051
>>277050>по-друге вони частково мають рацію, більшості людей шкільна математика після 9го класу в жизні буде не потрібнаТоді в нас не буде високотехнологічних підприємців та крутої інженерної хуйні. Якщо ті люди переконали себе у тому що математика їм не потрібна, то якщо їм пояснюватиме суспільство що математика це різниця між довбойобом та не довбойбом то вони одразу ж підуть штудіювати усе що треба.
Я розумію твій академічний підхід але це все треба проганяти крізь призму соціуму та ієрархічної гонитви щоб досягти реальних масових звершень.
Хуйні написав на Кропиві, день пройшов продуктивно
⋮⋮⋮
No.
277052
>>277051Я не впевнений що розповсюдженість скепсису про корисність математики в житті серед населення дуже сильно впливає на рівень високотехнологічних підприємств, бо цей скепсис загальносвітовий, за деякими виключеннями. Думаю що зараз ботлнек це кількість регуляцій які накладають на підприємства.
Але навіть якщо й так, то в мене нема внутрішнього бажання таку книжку писати, бо я сам з прикладами "використання математикі в житті" середнє ознайомлений, а якщо нема бажання то книжка вийде поганою, або не вийде взагалі. Краще напише таку книжку якийсь фізик або інженер.
⋮⋮⋮
No.
277060
>>277051> математика це різниця між довбойобом та не довбойбомЗ чого ти взяв?
Ніколи в житті не бачив математика-довбограя?
Контролюй свою меншовартість.
⋮⋮⋮
No.
277069
>>277060Є таке, рос матики, дуже відомі, з міжнародними преміями, з теоремами які сильно вплили на ту чи іншу область, всираються за рашку як можуть, пишуть пости на фейсбуку про українців-нацистів англійською "для колег", при тому ще й напівспокійно можуть відвідувати міжнародні конференції европейські/американські, або працювати в японії. Це пизда. У людей загалом емпатія дуже обмежена, нацистів ненавидили через те що німеччина заїбала пів-европи, а рус заїбали тільки українців, тому їх ненавидять тільки українці, а для европейців то "всеж ми люди" і "у кожного своя політична позиція, а наука внє паліткі".
⋮⋮⋮
No.
277070
>>277060Це я про теоретичне брендування необхідності математики для мас.
>>277052Ну добре тоді.
>>277069Універсальний контраргумент це реально ксенофобні руске марші і те що з себе представляє їх правацтво. А так да, будь які гіперпрофесіонали у своїй індустрії скоріше за все будуть не дуже просунутими в інших сферах. Це як прокачувати персонажа на один скіл.
⋮⋮⋮
No.
277072
Не переконає нікого, бо ультраправі вони всюди однакові. Про "скоріше за все" не впевнений, та й тут не зовсім про недораскачені інші скіли. Але то таке, не хочу заглиблюватись.
⋮⋮⋮
No.
277083
>>277033Мене більше цікавить як люди опиняються на цьому шляху з дитинства? Фіз-мат ліцей мабуть і потім теор математика у ВНЗ (український)?
До цього питання: чи можливо взагалі добре вивчити математику (або взагалі будь-що комплексне) самостійно НЕ будучи в колі однодумців протягом багатьох років життя?
А то я вже останніх років 4 топчусь на місці як довбойоб намагаючись хоча б заповнити пробіли, які, по-ідеї, колись проходив, але ніхуя не засвоїв бо то була технічна нематематична спеціальність, де загалом настрої були такі що можна й без цього.
Чи якщо справді хочеш то шлях тільки в універ?
⋮⋮⋮
No.
277086
>>277083Ні в мене нестандартний шлях (по якому я доволі недвозначно деанонюсь, але піхуй вже), в мене в школі вчителька тренувала на олімпіади з інформатики з 7 класу, тому що в 5 написав в гуглі "як створити сайт" і вивчив хтмл, ксс і трохи паскаля. Після школи поступив на прогера але швидко усвідомив що програмування то хуйня для задротів тупих (вибачте прогери люблю вас дуже), і від олімпіад по інформатики там нічого нема. Тому самоосвітнювався математикою і в магістратуру йшов вже на "математику і мат.фізику", а зараз пхд по чистой роблю. А якщо ще глибше, то мені завжди подобалось дві речі: загадки і інтелектуальна краса, через то в мне й гарно з олімпіадами по програмуванню й пішло, тому що це інтелектуально красиві загадки (іноді). В звичайному програмуванні першого нема by design, хотів знайти друге але зрозумів що і другого нема. Це трохи simplified версія мого життя бо були серйозні пертурбації. Таке.
>До цього питання: чи можливо взагалі добре вивчити математику (або взагалі будь-що комплексне) самостійно НЕ будучи в колі однодумців протягом багатьох років життя?Складно сказати. Скоріш ні чим так з моєї точкі зору. Але знаю контрприклади. Я й сам не був в колі однодумців, всі мої колишні одногрупники стали програмистами (буквально всі).
>Чи якщо справді хочеш то шлях тільки в універ?Ще репетитора можна знайти напевно, при тому не обовʼязково два рази на тиждень, але розмова раз на місяць (хоча б) потрібна.
⋮⋮⋮
No.
277096
>>277086> Тому самоосвітнювався математикою і в магістратуру йшов вже на "математику і мат.фізику"> Я й сам не був в колі однодумцівІ як це в тебе було? Скільки треба було вйобувати годин на тиждень, особливо враховуючи паралельно основну діяльність? Коли вступив на магу то не відчував себе відстаючим?
>>274897> на більш серйозну репититори здебільшего не потрібні.>>277086> Ще репетитора можна знайти напевно, при тому не обовʼязково два рази на тиждень, але розмова раз на місяць (хоча б) потрібна.Так як, куди? Знаєш реальні випадки як репетиторство відбувається в 23-24+? А то я тільки краєм вуха чув що такі існує, але що і як то незнаю.
Будеш моїм репетитором? Колись в 11-му класі ходив на підготовчі курси до ЗНО в групу з 4-6 чоловік (коли як приходило), де було 25% теорії, 75% ми розв'язували задачі, і додому було повторення + все, що не встигли розв'язати на занятті.
Досі згадую цей період як найкращий в моєму житті з точки зору продуктивності і результату. Ось думаю як цей досвід повторити, коли ти вже старий
⋮⋮⋮
No.
277097
>>277096Це був несистемний нонстоп, тобто приходив з пар і читав математику нонстопм, іноді шкодячи навіть основній програмі, дещо з того що вчив не знадобилось зовсім. Ні, в цілому відчував що по деяким топікам знаю навіть більше.
>Так як, куди? Знаєш реальні випадки як репетиторство відбувається в 23-24+? А то я тільки краєм вуха чув що такі існує, але що і як то незнаю. Будеш моїм репетитором?Не знаю якщо чесно, чув що таке існує. В теорії я міг би спробувати, але не впевнений що воно ок для тебе буде бо не мав такого досвіду раніше.
⋮⋮⋮
No.
277098
>>277070> Це я про теоретичне брендування необхідності математики для мас.А яка необхідність? Нема її.
В 95% випадків математика рівня похідних і вище нафіг не потрібна.
Навіть програмістам, бо переважна більшість з них вже не пише мікроконтролери.
Зараз дрочити на математику це наче дрочити на ті ж книги: пережиток минулого без реальної кореляції зі статусом сьогодні.
Видихуй.
⋮⋮⋮
No.
277109
>>277098Математика вище похідних не потрібна мавпам, які після піврічних курсів дрессирування пишуть фронтенд, крудню та інтерпрайс. І в програмуванні мікроконтролерів вона також не потрібна.
А ось для тих хто на передньому краї розробляє нові технології ще й як потрібна, наприклад в блокчейні, ші, компіляторах, формальній верифікації, квантових технологіях
⋮⋮⋮
No.
277110
>>277097> приходив з пар і читав математику нонстопмЩо, тупо теорію читав? Завдання там вирішував? Як розумів чи рухаєшся в правильному напрямку?
Як до речі ставишвся до модних сьогодні онлайн-курсів? Coursera, eDX, OCW; інтерактивні платформи типу Brilliant (якщо чув взагалі), Khan Academy?
Я коли літературу намагаюся читати без достатнього бекграунду по темі, то натикаючись на щось незрозуміле тупо впадаю в ступор і далі не рухаюсь. З відеоматеріалами відчуття трошки краще, бо воно не зупиняється незважаючи на твою увагу. Але потім все рівно доводиться перемотувати і така ж хуйня виходить. Тому я й прийшов до висновку що потрібен якийсь спосіб отримувати фідбек і мати змогу рухатися далі навіть якщо поки щось не зрозуміло.
На вищезгаданих онлайн-курсах, фідбек на форумах ну просто дохуїще часу та сил займає, і жодних гарантій що ти отримаєш саме ту відповідь, на яку очікував.
> я міг би спробуватиТи зараз серйозно? Для анона? Для мене?! :3
⋮⋮⋮
No.
277111
>>277109> наприклад в блокчейні, ші, компіляторах, формальній верифікації, квантових технологіяхБез зрадойобства, але так кажеш наче у нас щось з переліченого роблять.
⋮⋮⋮
No.
277113
>>277097>>277110О, ще як ти ставишся до шкільної/університетської олімпіадної математики? Чи допомагає знайомство з нею в розумінні стандартного курсу (диф./інт. числення, лінійна алгебра, диф рівняння, теорія ймовірності і статистика)? Чи радив би дорослив взагалі дивитися в бік шкільної математики (як початок)?
А також роль формальних доведень та визначень всього і вся? Мені здаються ці речі важливими щоб розуміти на що ти натикаєшся, коли ти на це натикаєшся (і воно не в підручнику, де явним чином вказана тема). Чи достатньо пояснень з аналогіями і "простими словами"?
⋮⋮⋮
No.
277115
>>277111Роблять наші, але не для нашої користі. В цьому звісно проблема, що ми не можемо організувати щось ґрутновно.
І це не дивно, якщо на дошці об'яв в холі в університеті у нас написано не "добре вчися щоб запустити на марс українську ракету" а "добре вчися щоб з'їбати звідси працювати на німців"
⋮⋮⋮
No.
277119
>>277110Типу того, щось типу "прочитати весь підручник Х і перерішати всі вправи в ньому" правильного напрямку особливо не існує, якщо кудись рухаєшся то кудись придеш. Ступори це нормально, вміння їх правильним чином опрацьовувати це один з підскіллів навчання (я досі не вмію). До платформ і олімппіад ставлюсь гарно. Олімпіади трохи допомагають в розумінні стандартних курсів, але опосередковано. Якщо шкільної математики зовсім не знаєш то звісно треба почати зі шкільної. Формальні доведення і визначення то і є математика, те що відокремлює математику в окрему діяльнітсь і відрізняє її від, скаєімо, фізики, тому роль їх в вивченні математики величезна. Аналогії і прості слова це щось що краще допомогає зрозуміти визначення і доведення, теж потрібні (див
https://terrytao.wordpress.com/career-advice/theres-more-to-mathematics-than-rigour-and-proofs/).
⋮⋮⋮
No.
277122
>>277119> якщо кудись рухаєшся то кудись придешТобто це загалом і був твій підхід, тупо їбашити в великих масштабах? Я намагався так робити, але як воно буває з будь-яким заняттям, зробиш перерву декілька тижнів/місяців і щось вже й забув що ти вчив. Чи загалом йдеш по матеріалу неперервно, ніби все ок, а що ти там конкретно вивчив тиждень два тому вже й не згадаєш. І ось ці моменти просто піздєц discouraging. Ось так і топчусь на одному місці, як згадував з самого початку.
> Якщо шкільної математики зовсім не знаєш то звісно треба почати зі шкільної.Тут як сказати, шкільну математику я як раз знаю доволі таки непогано. Те ж ЗНО здав на 200 балів (в 15-му році там був поділ на базовий і просунутий рівень, то я довбойоб здавав тільки базовий бо вступав на НЕ мат спеціальність). Але то більше механічні знання, як і з університетським курсом. На доведення ні в школі, ні в універі уваги особливо не звертав. Тому часто не знаю звідки я "знаю" певні речі і чому вони правда. Я правила диференціювання стандартних функцій за таблицею вивчив раніше, ніж дізнався про те, що вони насправді виводяться в типових курсах числення (які я потім дивився в лекціях англійською). І ситуація така, що задачку якусь ти можеш вирішити, а ось природи того що робиш взагалі не розумієш чи погано розумієш. Ці всі пробіли дуже сильно зайобують. Тому я й питав про олімпіадну математику, бо мені здається що саме там є менше механічної роботи, а як раз більше пояснень суті та доведень.
Статтю прочитав, цілком погоджуюсь. Я ось зараз знаходжуся в не достатньо опанованій першій фазі, але по деяких темах відчуваю що міг би перейти до другої. Навіть була спроба, але блядь, як же ж це складно.
І зараз, як Теренс пише, часто речі здаються якимось нонсенсом, коли нагаєшся думати про їх властивості без rigorous thinking.
Ось такі справи. Як прогресувати так щоб без
пекельного борошна прогризання через 95% помилок в навчанні навіть і не знаю.
⋮⋮⋮
No.
277123
>>277122>Тобто це загалом і був твій підхід, тупо їбашити в великих масштабах? Я намагався так робити, але як воно буває з будь-яким заняттям, зробиш перерву декілька тижнів/місяців і щось вже й забув що ти вчив. Чи загалом йдеш по матеріалу неперервно, ніби все ок, а що ти там конкретно вивчив тиждень два тому вже й не згадаєш. І ось ці моменти просто піздєц discouraging. Ось так і топчусь на одному місці, як згадував з самого початку.Там багато деталей які я не розповів через які мій досвід не стовідсотково релевантний для тебе. По-перше я вже не з повного нуля був, бо яку-неяку базу по аналізу і лін алгебрі університет дав. По-друге я знайшов собі в університеті наукового керівника математика якій керував мою освіту, і ще іноді ходив на семінари в київський інмат. По-третє в мене був загалом сильний бекграунд з олімпіад з інформатики, скажимо, я гарно знав теорію графів і елементарну теорію чисел (краще ніж деякі мої викладачі іноді). Але в цілому так, більше було щось типу "хочу вивчити цей топік бо він дуже абстрактний а значить дуже крутий і красивий" (молодим був ще не мав смаку до математики і вчив все що мені казалось абстрактним і крутим).
> Те ж ЗНО здав на 200 балівВітаю, навіть я на 190 наскількі памʼятаю (переплутав радіус з діаметром в якомусь завданні). Тоді тобі точно не треба вчити шкільну і починай одразу з університетської.
> Тому я й питав про олімпіадну математику, бо мені здається що саме там є менше механічної роботи, а як раз більше пояснень суті та доведень.Там дуже специфічні завдання які націлени на нестандартне мислення і трюки, щось типу компʼютерної гри baba is you, а не про звʼязки з математикою як наукою. Це не значить що це щось погане, хоча. (Гарні) університетські підручники вони теж про розуміння і "як воно все працює", і навіть вони більше про це ніж олімпіадні задачі.
⋮⋮⋮
No.
277131
>>277125Теж тіко створив. Твій лінк не працює, має бути хендл#чьоторьохзначний код.
⋮⋮⋮
No.
277239
>>274298Математика відкрита чи винайдена?
⋮⋮⋮
No.
277240
З незрозумілих зараз причин з'ясувалося що вітчизняна вікі
https://uk.m.wikipedia.org/wiki/Аксіоми_Пеано визначає одиницю як перше натуральне число. Англомовна каже що це нуль
https://en.wikipedia.org/wiki/Peano_axioms (за десять років Пеано змінив думку). А чому полягає зрада?
⋮⋮⋮
No.
277243
>>277240Зараз стандарт в арифметичних формальних системах включати 0, різниці техничної особливо нема, наскільки я розумію.
>>277239Думаю що винайдена, але є різні точкі зору.
⋮⋮⋮
No.
277244
>>277240Вітчізняну вікі можна редагувати до речі!
⋮⋮⋮
No.
279023
>>274298Як здати зно з математики, якщо я повний дебіл, і ніхуя не вчив десь класу з 6?
⋮⋮⋮
No.
279024
>>279021Привіт!
>>279023Спробувати щось вивчити за півроку.
⋮⋮⋮
No.
279796
>>277033>Докотився природнім шляхом: якщо хочеш щоб за чисту математику давали грощі (я хочу) то ти залишаєшся в академії, особливо нема опцій інших.Чиста математика це така математика, яка зараз немає практичних застосувань? А який сенс вибирати саме цю частину від математики? Вона більш складна чи цікава?
⋮⋮⋮
No.
279813
>>274298За програмування шариш? Наскільки важливо добре знати матан, щоб вкотитися у ойті?
⋮⋮⋮
No.
279814
>>279813Матан допоможе стати крутим програмістом, але вкотитися і працювати в ойті можна взагалі без нього.
повз не оп
⋮⋮⋮
No.
279820
>>279814взагалі не важливо
матан треба виключно якшо в тебе сфера вимагає матану
програмування це інженерія
тобто в першу чергу треба уміти з купи детальок
складати саме ту штуку яка вирішує саме твою проблему і твою задачу
математика це лиш один з інструментів (детальок)
який можна використати при бажанні
а можна і ні
звісно краще волоіти інструментом чим не володіти
але то нюанси
⋮⋮⋮
No.
279863
>>279796Так, більш складна і цікава. І взагалі інша діяльність, окремо від прикладної: доводити теореми, а не мат.моделювати щось чисельно на компі.
>>279813Трохи знаю, згоден з двома попередніми дописувачами.
⋮⋮⋮
No.
279866
Клацав калькулятор у телефоні і там було Х!. Я паонатискав, помітив що воно перемножае числа якось. А ось нашо воно це робить, я не зрозумів. Що це за штука така?
⋮⋮⋮
No.
279877
>>279875Вибачте за подвійний допис, йобані компи знову роблять як не мають.
⋮⋮⋮
No.
280256
А як ота хуйня коротше називається коли матриці обчислюєш то шо воно коротше його треба знайти ну шоб потім помоножити на якийсь там добуток певних членів і шось воно коротше буде?
⋮⋮⋮
No.
280280
>>280256Визначник і союзна матриця.
⋮⋮⋮
No.
280747
>>274298Почитав нитку і здивувався, що можливо стільки всього знати, жесть. Скільки тобі років?
Нещодавно гортав інженерний підручник, дуже горіло від того, що більшість методик давалось без якихось пояснень, просто "робіть так, математики розберуться". Хотілося б трішки шарити, бо на перетвореннях лапласа я залип, на жаль(
Тому саме питання.. є якісь типу базові книжки, після освоєння котрих можна розуміти математику?
⋮⋮⋮
No.
280795
>>28074727. Дякую, я знаю обʼєктивно середнє, просто зовні завжди здається що багато. Дивився нещодавно підручники для початківців: Abbott "Understanding Analysis" та Axler "Linear algebra done right" мені здалися найкращими. Перетворення Лапласа мотивують зазвичай одним з двох способів: інтерпретація через операційне числення, гарно написана тут
https://math.stackexchange.com/a/105837 і інтерпретація як неперервні ряди Тейлора гарно пояснена тут
https://ocw.mit.edu/courses/18-03-differential-equations-spring-2010/resources/lecture-19-introduction-to-the-laplace-transform/
⋮⋮⋮
No.
281845
Через які системи аксіом можна робити якісь міркування про різні системи аксіом, якісь їх співвідношення і особливості, тобто які є аксіоматичні системи для роботи з аксіоматичними системами? Наприклад, якщо треба довести, що виходячи з одним набором аксіом якась теорема не може бути доведеною, але якщо додати одну конкретну аксіому, то довести ту теорему вже можна, які потрібні аксіоми для такого аналізу? Як називається цей розділ математики?
⋮⋮⋮
No.
282308
>>281845Будь-яка гьоделевська підійде, наприклад: PRA, PA, ZFC, Q, тобто будь-яка в якої можно формалізувати примитивно-рекурсивну роботу зі строками.
>Як називається цей розділ математики?Reverse mathematics, або, якщо більш загально, то теорія доведень.
>>281953Найкраще що є.
⋮⋮⋮
No.
282309
>>282308Приємно тебе бачити на дошці, вітаю!
⋮⋮⋮
No.
282312
>>282309Забув головне: дякую, друже!
⋮⋮⋮
No.
282319
>>282310>>282312Та я довбень шось вирішив Кропиву почитати вночі.
Твій тред мені взагалі нагадав що треба якось почати займатись оціма всіма математичними приколами і спробувати їх застосовувати у роботі чи взагалі знайти собі шось нове.
Люблю цей тред.
⋮⋮⋮
No.
282323
>>282321Плюс математика і усі ці цифри це не лише косінуси та інше але і статистика, моделі...та взагалі дофіга усього.
Я вже писав здається у цьому треді що на мою думку математика на жаль дискредитована поганими вчителями і що якщо люди хочуть реальної розбудови країни та тупо грошей то треба тупо вбухувати міл'ярди у якісну освіту.
Єх.
⋮⋮⋮
No.
282326
>>282325Боронь Боже, не кажу що всі вчителі погані, але оце
>загальна освітав дуже немотивованому стані. Ну і та сама стігма стосовно математики та математиків так.
Не хочу повторюватись у треді, але дуже кортить понити про це все, кек.
⋮⋮⋮
No.
282334
>>282326Велика частина того матеріалу що подається в школі - включно з математикою - не потрібна.
Школа і універ - це про дресуру, не про освіту.
⋮⋮⋮
No.
282335
>>274472Знайшов графіки Філіпа Хаґара Сміта, що трансформують плоский простір (і об'ємний теж) таким чином, що в розтягнутих областях виникають більш деталізована мапа з частинами всередині кола.
Використовуються вони для одночасного відображення кількох параметрів, включно з імпедансами (комплексний опір), адмітансами (комплексна провідність), коефіцієнтами відбиття, параметрами розсіювання, колом коефіцієнта шуму.
⋮⋮⋮
No.
282336
>>274298Скільки буде 150+150?
⋮⋮⋮
No.
282774
>>274298Як знайти площу криволінійної трапеції? А якщо цю криволінійну трапецію зробити тривимірною фігурою, то як тоді її обчислити?
⋮⋮⋮
No.
282873
>>282871Аксіоми—це пропозиції у формальних системах, які вважаються самоочевидно істинними.
Таким чином, вони не потребують жодних доказів, вони просто вважаються істинними апріорі.
Прикладом може бути «P [передумова] v [логічно пов’язана з] ~P [не передумовою]», поширена аксіома у
https://www.wikiwand.com/uk/Числення_висловленьГіпотези—це пропозиції у формальних системах або емпіричні наукові твердження, стверджувані на основі спекуляцій або припущень.
Мета полягає в тому, щоб визначити їх істинність чи хибність на основі дедуктивного чи індуктивного доказу, підтвердженого тестами та експериментами.
Молекули та кварки були гіпотетичними сутностями в науковій теорії, поки фактичні емпіричні випробування та експерименти не підтвердили їх існування.
Сьогодні темна матерія і темна енергія є гіпотетичними сутностями, які чекають емпіричного підтвердження.
https://www.quora.com/How-does-an-axiom-differ-from-a-hypothesis
⋮⋮⋮
No.
282874
>>282873* v [логічно відділена від]
⋮⋮⋮
No.
282876
>>282873>Таким чином, вони не потребують жодних доказів, вони просто вважаються істинними апріорі.>Гіпотези—це пропозиції у формальних системах або емпіричні наукові твердження, стверджувані на основі спекуляцій або припущень.Я не бачу тут відповіді на питання "Чи можна замінити всі аксіоми гіпотезами (передумовами)? Якщо так, навіщо взагалі потрібні аксіоми в формальних системах?". Може я невдало сформулював.
Всі аксіоми в усіх теоремах, які на них спираються, можна механічно замінити такими ж самими припущеннями ? Тобто якщо з аксіомами доказ чогось виглядав би "Виходячи с аксіом А та Б твердження В вірно", то можна це переформулювати в "Якщо твердження А та Б вірні, твердження В вірно". Якщо так можна робити, то навіщо потрібна додаткова сутність "аксіома"?
⋮⋮⋮
No.
282899
>>282876Відповідь може залежати від цілі.
Об'єктивно існують злічувані та нескінченні речі.
Злічувані речі можна порахувати в десятковій система числення, а можна у двоїчній, можна використовувати арабські цифри, а можна римські і т.д.
Аксіоми в системі знань мають ніяким чином не суперечити одна одній, бо система має функціонувати та бути зрозумілою.
Гіпотези, можуть бути в деяких моментах суперечливими, деякі можна перевірити та підтвердити, що перетворить їх на теорії.
Гіпотези можуть бути:
а) правдою
б) помилкою
в) правдою чи помилкою, але не відомо що саме
г) ані правдою, ані помилкою
ґ) або (в), або (г), але не відомо що саме.
Аксіоми ж є складовими "жорсткого каркасу" систем.
Якщо їх замінити несуперечливими (!) гіпотезами, то система недосконала, знаходиться у стані "творчої асиметрії".
⋮⋮⋮
No.
283078
>>282774Інтеграл, або подвійний інтеграл функції відстані точки на шапці трапеції (трапецоїда) до основання. Це визначення площі (обʼєму) криволінійної трапеції (криволінійного трапецоїда), або щось близьке до визначення. Є доволі багато техник для символьного і чисельного обчислювання інтегралів, деякі з них частково проходять навіть в школі.
>>282873Якщо коротко: різниці особливої нема і так зробити можна. Більш того, в деякому сенсі так і робиться, коли пишуть щось накшталт “T |- P”, то це можна читати двома способами: як “з (множини) гіпотез T можно вивести P” або “з (аксіом) теорії Т можна вивести Р”. Якщо довго, то є деякі нʼюанси. Якщо вони заплутають тебе ще більше, то ігноруй їх і сприймай тільки попереднє речення як повну відповідь, можливо що на твому рівні розуміння про них думати не треба:
1) Аксіоми бувають трьох типів “логічні закони”, “аксіоми теорії” та “аксіоми які є частиною визначень”. “Логічні закони” їх має сенс сприймати як частину дедуктивної системи під назвою “касичне предикатне числення першого порядку гільбертовського типу з =”. “Логічні закони” треба сприймати на тому ж рівні що і правила виводу, що і рекурсивні визначення формул і термів, що і рекурсивне визначення коректної підстановки, що і визначення алфавіту формальної мови. Це не просто питання стилю, якщо цього не робити то, наприклад, теорема Гьоделя про повноту просто не вірна (або треба оговорювати що вона вірна тільки для теорій або множин гіпотез які містять всі логічні закони). “Аксіоми які є частиною визначень” це синтаксичний цукор і можна взагалі без нього. Коли я говорю “аксіоми” я маю на увазі саме “аксіоми теорії” (наприклад: аксіоми планіметрії Тарського
https://en.wikipedia.org/wiki/Tarski%27s_axioms). Інакше, “логічні закони” не можна сприймати як гіпотези.
2) Є деяка трудність з параметрами, розглянемо “x - ділиться на 4 |- x - парне”, це нормальне твердження про умовний вивід. Але не є нормальним казати що “з системи аксіом аріфметики до якої додана аксіома “x - ділиться на 4” виводиться “x - парне””. Тому що аксіома “х - ділиться на 4” це теж саме що аксіома “всі числа діляться на 4”, її додавання робить аріфметику протиречивою, бо 1 не ділиться на 4. Інакше, гіпотези с параметрами не можна сприймати як аксіоми. Або можна, але обережно, спершу замінивши всі змінні на додані до сигнатури формальної мови константні символи. Якщо ми забороняємо гіпотези та теореми з параметрами то цієї проблеми нема.
3) Слово "гіпотеза" є в двох сенсах (більше ніж в двох, але не концентруємось на поганому). Є “гіпотези правила перетворення", ті що стоять нагорі від черти в Modus Ponens, а є "гіпотези умовного виводу", ті про які думаєш ти, коли наводиш приклад "з гіпотез А і В виводиться С". Це різні речі.
Перша це частина формальної системи, це правила гри, правило перетворення MP каже “якщо маєш формули “8 ділиться на 4”, “8 ділиться на 4->8 ділиться на 2” в свому зошиті, то тобі (по правилам гри!) дозволено дописати формулу “8 ділиться на 2” в свій зошит”. “8 ділиться на 4” та “8 ділиться на 4->8 ділиться на 2” є гіпотезами правила перетворення МР. Тут не можна сказати що це теж саме що з аксіом “8 ділиться на 4” та “8 ділиться на 4 -> 8 ділиться на 2” можна вивести “8 ділиться на 2”. По якому правилу можна вивести? По тому самому правилу МР, подставивши ці два твердження замість гіпотез МР. Інакше, гіпотези правил перетворення не можна сприймати як аксіоми.
Це відрізняється від твердження, що з аксіом “12 ділиться на 2", "12 ділиться на 3" можна вивести "12 ділиться на 6", бо в цьому випадку це не є застосуванням правила перетворення, а є металогічним твердженням про те що користуючись аксіомами аріфметики, логічними законами і правилами перетворення можно з деякіх двох тверджень вивести третє. І в цьому випадку є нормальним казати, що з аксіом аріфметики і аксіом "12 ділиться на 2", "12 ділиться на 3" можна вивести "12 ділиться на 6".
4) В деякіх формальних системах “гіпотези" або “аксіоми" є частиною числення, не металогічними поняттями. В численнях ґенценовського типа “гіпотези" є частиною системи. В цьому випадку зʼявляється третій сенс слова "гіпотеза", а саме, та послідовність формул що стоїть ліворуч від штопора в секвенті. Цей сенс сильно відризняється від двох попередніх. Наприклад, ліворуч від штопора в секвенті дозволено мати тільки скінчену кількість гіпотез, але коли ми кажемо про гільбертовське числення, то металогічний вираз Г |- phi має сенс навіть для нескінченних множин гіпотез Г. По приблизно тим самим причинам що і в 3, гіпотези в секвентах в численнях ґенценовського типу не можна сприймати як аксіоми.
Це, звісно, відноситься до математичної логіки, яка наука про формальні мови і їх семантику, в філософській лозіці, яка studies про те як правильно будувати аргументи, про яку частково писав схемний-кум вище, “аксіома” і “гіпотеза” мають свої сенси (напевно).
⋮⋮⋮
No.
283304
Сьогодні мене захопила ідея циклічності алгоритмічних операцій у математиці.
А також способи уніфікації, тобто споріднення, різноманітних систем...
Ось, наприклад,
https://www.wikiwand.com/uk/Гіпотеза_Коллатца :
1. Обираємо будь-яке число
2.1. Якщо воно парне, то ділимо навпіл
2.2. Якщо воно непарне, множимо на три та додаємо 1.
На другому зображенні числа підібрані таким чином, що в кожному рядку та стовпчику цифри не повторюються.
У статті про ізоморфізм є наступні корисні тези:
«Це як порівнювати яблука та апельсини. Графіки та групи не мають спільного словника. Отже, в таких ситуаціях слід створювати співвідношення.»
«У Шелаха є теорія,—сказав Ласковскі,—що певні запитання стають легшими, коли ви піднімаєте їх до вищої потужності (вищих рівнів нескінченності).
Тому що, коли числа стають дуже великими, відстань між важливими числами, такими як прості та квадрати цілих чисел, зростає.»
https://www.quantamagazine.org/mathematicians-solve-decades-old-classification-problem-20210805/Додатково можна розглянути:
https://www.wikiwand.com/uk/Бієкціяhttps://www.wikiwand.com/uk/Дерево Калкіна — Вілфаhttps://www.wikiwand.com/uk/Ряд_ФареяЦе було б корисно у розробці системи збалансованих характеристик (з паралельною приналежністю до різних типів груп) з унікальними значеннями, а також для створення дерев вибору типу
https://www.wikiwand.com/uk/Мінімакс для алгоритму ШІ.
⋮⋮⋮
No.
283802
>>283304Гарна стаття, не знав що [закреслити]Шелах ще живий[/закреслити], що ізоморфізм абелевих груп без кручіння є універсально борелевською множиною.
> Графіки та групиграфи та групи.
⋮⋮⋮
No.
283803
>>283802> універсально борелевською множиноюБорель-повною точніше.
⋮⋮⋮
No.
284315
>>282899В
https://www.wikiwand.com/uk/Теореми_Геделя_про_неповноту йдеться про те, що будь-яка аксіоматична (наприклад, математична) система ніколи не зможе доказати абсолютно усі правдиві твердження, що можуть бути знайдені всередині аксіоматичної мови цієї системи.
Іншими словами, можуть існувати такі правдиві твердження, коректність котрих неможливо довести, хоч вони і є такими.
Як, наприклад,
https://www.wikiwand.com/uk/Гіпотеза Гольдбаха .
⋮⋮⋮
No.
284333
>>284315Чому Гольдбаха не можна довести?
⋮⋮⋮
No.
284337
>>284333Тому що існує проблема неповноти математики, як казав Гедель:
"Одне з двох: або математика занадто обширна для людського мозку, або ж людський мозок являє собою щось більше, аніж просто машина."
Ну і ще, тому що "достатньо обчислювальна
https://www.wikiwand.com/uk/Ефективність_алгоритму формула для знаходження простих чисел ще невідома":
https://www.wikiwand.com/en/Formula_for_primesМаркус дю Сотуа гарно це сформулював: "Ми можемо мати труднощі щодо підтвердження цих речей, однак ми ще не розширили математичні аксіоми до такого рівня, аби вони піддавались доказуванню."
Gödel's Incompleteness Theorem - Numberphile
⋮⋮⋮
No.
284339
>>284337Ти щось неправильно зрозумів, невідомо те що гіпотезу Гольдбаха неможливо довести.
⋮⋮⋮
No.
285014
Підійму про всяк.
⋮⋮⋮
No.
285958
>>284337"Не існує абсолютної істини у математиці.
Математика є самореферентною, тому ми можемо доказати лише ті твердження, що стосуються самої математики.
Це не було великою проблемою, до поки не виникла
https://www.wikiwand.com/uk/Континуум-гіпотеза.
Якою б не була ця проблема, одна група людей покладала надії на те, що вона виявиться правдою, а інша група вважала, що це є фальш...
До поки не був знайдений доказ того, що насправді неможливо доказати, чи ця гіпотеза є правдою, чи навпаки, лише на підґрунті математичних аксіом.
Математик
https://www.wikiwand.com/uk/Пол_Джозеф_Коен:
"Тепер стало відомо, що правдивість або фальшивість Континуум-гіпотези не може бути визначена за допомогою сучасної
https://www.wikiwand.com/uk/Теорія_множин"."
The unsolvable problem that launched a revolution in set theoryhttps://www.wikiwand.com/uk/Теорія_множин_Цермело_—_Френкеляhttps://www.wikiwand.com/uk/Аксіома_Мартінаhttps://www.wikiwand.com/uk/Аксіома_виборуhttps://www.wikiwand.com/uk/Алгоритмічно_нерозв'язна_задачаhttps://www.wikiwand.com/en/Diamond_principlehttps://www.wikiwand.com/en/Forcing_(mathematics)
⋮⋮⋮
No.
285965
>>274298Зробиш контрольну роботу з алгебри для мого брата?
⋮⋮⋮
No.
285971
>>285958> Не існує абсолютної істини у математиці.До деякого ступеня. Математичні твердження фільтровані за рівнем абстрактності (analytical hierarchy), CH знаходиться на рівні Sigma_2^1 (дуже абстрактне). На самому низькому рівні, Delta_0^0 знаходяться твердження накшталт 2+2=4. Є модеруємий консенсус стосовно того, що має сенс говорити про абсолютну істиность аріфметичних тверджень, які знаходяться на рівнях Sigma_1^0 і нижче, бо вони в деякому сенсі дуже фізичні, стверджують що деякі программи зупиняються, або працюють вічно.
Стосовно truth value контінуум-гіпотези або більш абстрактних тверджень, дійсно є різні думки, які гарно підсумовані в статті Maddy "Believing the axioms I", або в нитці
https://mathoverflow.net/questions/23829/solutions-to-the-continuum-hypothesis>>285965Ні.
⋮⋮⋮
No.
285972
> Sigma_2^1
Sigma^2_1
> Sigma_1^0
Sigma^1_0
хто це буде перевіряти, хоча
⋮⋮⋮
No.
285978
>>274298В чому суть всієї цієї гомотопічної теорії типів?
Нащо абстругувати концепт "еквівалентності" і чи це допоможе перезагортати монадки на Haskell або доводити теореми на Agda чи Coq?
⋮⋮⋮
No.
285992
>>285982Дякую, не те що б я щось розумів, але принаймні частина про теореми з топології на пруверах мейк сенс.
Енівей, пісня із недалекого майбутнього:
Proof assistants get it done
But AI do it better
Proof assistants have their fun
But AI do it better
La La La La
Hey Hey
⋮⋮⋮
No.
286020
>>286007Так здається і роблять, тренують АІ генерувати пруфи в середені якогось пруф-чекера, таким чином якщо АІ згенерує пруф, то буде впевненість що це дійсно є пруфом, а не просто послідовністтю слів. Ось щось таке
https://arxiv.org/pdf/2210.12283.pdf наприклад.
⋮⋮⋮
No.
286024
>>285971Хм, абстрактність має ієрархію... Ця думка мені нагадує tier list улюблених речей/ персонажів, наскільки вони практичні, деталізовані тощо. Сама ієрархія може бути практичною, у якості платформи для вибудови цілісних систем, відповідно складові цих систем будуть дотичними одна до одної, і це має значення більше, ніж, так би мовити, шаблонна монолітність наборів множин. Тобто існує багатоплановий спектр нескінченної кількості значень. І якщо виокремити певні дихотомічні ділянки, можливо вони будуть навіть перетинатися та мати спільні елементи, то можливо виявити як закономірності всередині ділянок, так і смислове відображення в інших, за допомогою тих методів, що підходять... Тобто це можуть бути ієрархії, як систематизовані гроноподібні каталоги, а можуть бути ієрархії, що вказують на послідовну трансформацію елементів, конвертацію їхніх значень, що призводить до об'єднання, як абсолют.
https://www.wikiwand.com/en/Absolutenesshttps://www.researchgate.net/publication/222119409_Generic_absolutenesshttp://www.differencebetween.net/language/difference-between-dichotomy-and-paradox/https://www.wikiwand.com/en/Prewellorderinghttps://www.wikiwand.com/uk/Несуперечністьhttps://math.stackexchange.com/questions/499560/why-is-the-continuum-hypothesis-believed-to-be-false-by-the-majority-of-modern-s/499947#499947https://www.winterschool.eu/files/320-Solecki_Tukey_reduction_among_analytic_P-ideals_and_sigma-ideals_I.pdf>деякі программи зупиняютьсяhttps://www.wikiwand.com/uk/Алгоритмічно_нерозв'язна_задача>працюють вічноhttps://www.wikiwand.com/uk/Нескінченний_цикл
⋮⋮⋮
No.
286037
>>286024Можливо... В лозіці і теорії обчислювальності є багато ієрархій, це точно є правдою.
https://rmzoo.math.uconn.edu/diagrams/ як приклад.
⋮⋮⋮
No.
286039
Що таке число?
⋮⋮⋮
No.
286041
>>286039В математиці визначення "числа" нема, є тільки визначення специфікацій "натуральне число", "ціле число", "дійсне число", тощо.
З формалістськіх позицій, визначення даються в межах якоїсь формальної мови, стандартно, ZFC. Ця формальна мова і її аксіоми беруться за фундаментальні, невизначені, базові поняття. Натуральні числа в ZFC визначаються як найменьша індуктивна множина, ось тут можно побачити абсолютне формальне визначення множини натуральних чисел в ZFC:
https://us.metamath.org/mpeuni/df-om.html .
Але це все більше відповідь на питання "Як закодувати те що ми розуміємо під натуральними числамі в ZFC?", а не на "Що таке натуральні числа?". В книжці Hamkins "Lectures on the Philosophy of Mathematics" є така відповідь, я. з нею згоден.
⋮⋮⋮
No.
286092
>>286041"В кінці кінців, що таке число?
Я насправді дуже вибачаюсь, але ми не знаємо повністтю що таке число. Проблема математичної онтології досі не вирішена. Хоча в нас є багато філософських перспектив на це питання, з моєї точкі зору, ні одна з них не є цілком задовільненою. Ми насправді не знаємо повністтю що значить робити твердження про існування математичних обʼєктів."
⋮⋮⋮
No.
286093
>>286092Скільки тобі років? Ти ж не можеш відповвсти мені корова років. А от 15 років (наприклад) вже можеш.
⋮⋮⋮
No.
286096
>>286093Це правда звісно, а це аргумент до якої тези був?
⋮⋮⋮
No.
286097
>>286096 До тези, що визначення числа (кількості) досить чітко детермінується у порівнянні з іншими поняттями: слово, літера.
⋮⋮⋮
No.
287495
>>287492Вони чудові, дуже.
А "непрості" числа мають доволі неочікувані співпадіння:
⋮⋮⋮
No.
287497
>>287495Чудово що тобі подобається! Але єдина закономірність яку я убачаю в
схемі полягає в тому що вверх-низ і ліво-право співпадають, що пояснюється тим фактом що нижня пара 20,45 обрана таким чином, що є помноженою на 5 верхньою парою 4,9.
⋮⋮⋮
No.
287501
>>287497Дійсно, чомусь не звернув уваги, будь ласка, не зважай.
Складені числа мені подобаються у якості складових завдань по комбінаториці.
А ще інколи виходить не тільки розв'язувати, а й вигадувати простенькі завдання:
⋮⋮⋮
No.
287502
>>287501Теж люблю вигадувати щось просте, на свому рівні.
В 2) термінологія неправильна, правильно буде "Чому дорівнює довжина дуги кола (r=2, з центром в точці В) яку видно під кутом АВD?"
Твоє "окружність кола" є "колом круга" (скоріш за все, тут не впевнений).
Твоє "сегмент" є "дуга".
"Окружність кута" - так не пишуть.
В 3) даних недостатньо і відповідь залежить від кутів між цими відрізками.
⋮⋮⋮
No.
287575
>>287502Сподіваюсь так краще.
Ідея цього завдання прийшла на думку після перегляду короткого уроку про радіани та спроби створити шрифт із літерами на основі лише одної "супермісткої" геометричної фігури.
⋮⋮⋮
No.
287604
>>287576Я не дописую в цьому треді, але люблю вітати пана математика. Тож вітаю вас, пане математик!
⋮⋮⋮
No.
287617
>>287603Well enough. Хоча я б написав "сегмент круга".
>>287604Вітаю, пане.
⋮⋮⋮
No.
288480
Підійму.
⋮⋮⋮
No.
288485
>>288480Вітаю пана математика! Чи знаєте ви приклади людей які починали вчити математику після тридцяти чи навіть сорока років?
⋮⋮⋮
No.
288487
>>288486Цікаво, дякую за відповідь! На жаль не маю профільних запитань, але дуже тішить що ти відвідуєш борду.
⋮⋮⋮
No.
288492
>>288490Дякую, треба було самому загуглити, але мені цікавіше завжди чути anecdotal evidence.
Може якось додам тебе у Дискорді щоб не загубити дописувача.
⋮⋮⋮
No.
288499
>>288495>і ще уявити моторошно звідки ти його знати можешТи ж сам тут залишав
>>277125, лол. Я ще тоді хотів додати, але забувсі.
⋮⋮⋮
No.
288501
>>288500Було б смішно якщо я додав б того чувака і думав би що то ти.
⋮⋮⋮
No.
288772
1, 2 не зовсім про залишкі, це про скінченність. Якщо в тебе скінченне число станів і ти робиш одну й ту саму операцію знову і знову, яка переводить якийсь стан в якийсь інший, починаючи з якогось стану, то вона колись зациклиться. Подобається те що тебе то так зацікавило що навіть в 3 виписав всі залишки в стобчик. Відео гарне, теж дивився. Ще що можу сказати про 3:
Во всіх стовпчиках дробна частина повторюється з періодом в знаменник через те що з рац. дроби можно виділити цілу частину.
7/7 = 1
8/7 = 1 + 1/7
9/7 = 1 + 2/7
...
13/7 = 1 + 6/7
14/7 = 2
15/7 = 2 + 1/7
...
тому наприклад в першому стопчику кожні 7 кроків ціла частина буде збільшуватись на 1, а дробна повторюватись в періоді. Во всіх інших стовпчиках аналогічно.
Перший стовпчик повʼязаний з цикличними числами. період в 1/7, 2/7, ..., 6/7 зміщується по циклу тому що 10 є генератором мультипликативної групи поля Z/7Z. Треба трохи розуміти елементарної теорії чисел, але грубо кажучи, це повʼязано з тим що 1, 10, 100, ..., 10^6 дають всі можливі ненульові остачі при діленні на 7, у той час як 1, 10, ..., 10^5 не дають всі можливі ненульові остачі по модулю 6, нема остачі 2 наприклад.
У другому стовпчику кожен дріб має скінченний запис тому що 8 = 2^3, а 2 є дільник 10 (10 база системи числення). Теж саме буде з 1/2^n5^m для будь яких n,m.
У третьому стовпчику є більш загальний факт: якщо щоб записати дріб в перійоді достатньо поділити число на достатнью кількість девʼяток. Наприклад: 1537/9999 = 0.(1537) це тому що права частина за визначенням є
0.15371537... = 0.1537 + 0.00001537 + ... =1537/(10000)+1537/(10000)^2 + 1537/(10000)^3 + ...
тому що поділити на 10000 це змістити групу цифр 1537 на 4 позиції праворуч. Сума нескінченної геометричної прогресії дає 1537/9999.
У четвертому стовпчику нічого незвичайного. Період довжини 2 тому що 10^2 mod 11 = 1
⋮⋮⋮
No.
288774
>>288772Так, основні моменти щодо
https://www.wikiwand.com/uk/Ділення_з_остачею розумію.
Зацікавили раціональні вузли:
https://www.wikiwand.com/en/Tangle_(mathematics) .
"Цікавою властивістю раціональних клубків є те, що еквівалентність між ними можна показати шляхом обчислення їх
https://www.wikiwand.com/uk/Ланцюговий_дріб."
"Якщо ці два неперервні дроби еквівалентні, то два раціональні клубки мають бути еквівалентними."
https://rationaltangle.wordpress.com/what-are-tanglesrational-tangles/
⋮⋮⋮
No.
289429
Чи існуе аналітичне рішення у рівняння x^5+x^2-1=0 ?
⋮⋮⋮
No.
289432
>>289429В радикалах ні. Група Галуа S5, тому за теоремою Абеля нема формули в радікалах. Дійсно, по модулю 2,3 многочлен факторізується на многочлени зі ступенями (5),(2,3) відповідно. За теоремою Дедекінда в групі Галуа містяться перестановки з циклічним типом (5) і (2,3), однак будь які дві такі перестановки породжують всю S5. Дійсно, перестановка типу (2,3) в кубі це перестановка типу (2,1,1,1) тобто транспозиція, а транспозиція і зсув породжують S5.
Якщо дозволити також користуватися деякими трансцедентними функціями (тета-функціямі якобі, або радикалом брінга), то будь яка квінтіка вирішується аналітично, зведенням до форми Брінга-Жирара, в тому числі і ця.
⋮⋮⋮
No.
289488
>>274298йоу, ескперт, з математики, ким працюєш? Я от ІПСУ з Богданським закінчив, але поклав хуй на математику як раз в другому семестрі третього курса й пішов в айті. Розкажи про свій досвід, може я зараз пожалкую, що поклав хуй і не їбашив матьошу далі?
Буде дуже прикро, якщо ти якийсь МЛ-єр чи ДС-єр, бо дуже якийсь неоригінальний вибір життєвого шляху.
⋮⋮⋮
No.
289490
>>289489Сорі, полінився почитати нитку перед тим, як задати своє попереднє очевидне запитання. А як у тебе стоїть питання з грошима та матеріальним забезпеченням? І з амбіціями? Я знаю, що багато олімпіадників в університеті і школі були амбіційними та високомірними хлопцями, та й я сам такий був. Через це ще й ідеалізовано пішов на мат. спеціальність, думаючи, що оволодію окультними знаннями та буду застосовувати математику до різних процесів у житті та таким чином отримувати профіт з усього. Звісно, то все не так. Ти писав, що тобі подобається інтелектуальна краса математика, тобто ти більше ідейно цим захоплюєшся цим, як річчю в собі, ніж як інструментом, який приносить для тебе особисту користь? І в чому полягає ідея ставати науковим співробітником, ти ж по факту задаром будеш передавати свої знання іншим амбіційним людям, які зможуть реалізувати їх та використати в своїх цілях без тебе. Це дуже альтруїстично, ти хочеш цим займатись виходячи з цих міркувань?
⋮⋮⋮
No.
289492
>>289490Мені зараз платять дуже добре, з моєї точкі зору. Так, більше ідейно, прагматики ніякої немає, навпаки, заняття математикою мені багато разів в житті заважало. Ідея в тому щоб отримувати грощі за математику, і не менеджити прибуток окремо вторічними зайнятосттями. Про амбітних людей які будуть використовувати розроблені мною знання - це ефективно неможливо, переважна (98%) частина чистої математики не придатна до того щоб використовувати її на практиці, і навіть щоб мати можливість читати мої дослідження потрібно бути особливим чином натренованним математиком, амбітні люди не мають належного тренування, з першої причини (неефективність чистої математики для "реалізації цілей", щоб це не значило).
⋮⋮⋮
No.
289493
>>289492Дякую за відповідь. Принаймні тепер я набув спокою, щодо того, що не дарма залишив цю справу. У мене така підозра і була, що далі залишаються тільки ідейні й немає сенсу беззмістовно себе цим мучати .
⋮⋮⋮
No.
289494
>>289493То є так, нема за що.
⋮⋮⋮
No.
289616
Навіщо є так багато сучасних аксіоматик евклідової геометрії (аксіоматика Гільберта, Бахмана, Тарского...) та чи зводимі вони одна до одної? Наприклад, чи можна аксіоми з однієї такої аксіоматики довести як теореми через іншу аксіоматику, та навпаки? Чи всі вірні/невірні/недоказові твердження в одній аксіоматиці будуть такими ж в іншій?
⋮⋮⋮
No.
289634
>>289492> скарлет консорциумаЧому зображення не достатньо українське?
⋮⋮⋮
No.
289653
>>289634Випадково, не бийте.
>>289616Особливо не навіщо не потрібно, вони всі не те щоб і дуже сучасні, з початку 20 сторіччя, коли логічні питання були модними (кризис основ і тп), зараз це доволі маргінальний топік на моє відчуття, хоча знаю тих хто займається аксиоматизаціями геометрії.
>та чи зводимі вони одна до одної? Наприклад, чи можна аксіоми з однієї такої аксіоматики довести як теореми через іншу аксіоматику, та навпаки? Чи всі вірні/невірні/недоказові твердження в одній аксіоматиці будуть такими ж в іншій? Так, так, так. За модулем техничних деталей, а саме:
1) Вони над різними сігнатурами (різні елементарні поняття), тому треба спочатку їх інтерпретувати одна в одній перш ніж порівнювати, але вони майже канонічно біінтерпретуєми, що неформально значить що ці теорії однакові.
2) Можна обрати в кожній записувати першопорядкову схему аксіом повноти, чи другопорядкову аксиому повноти. Другопорядкові теорії не порівнювальні з першопорядковими. Класично в Гільбертовській формалізації другопорядкова аксіома, а в формалізації Тарського першопорядкова схема, але це не важливо, можна і навпаки зробити.
Сучасний підхід це моделювати геометрію всередині ZFC фреймворка (теорії множин), в якому моделюється вся математика.
А ще більш сучасний не думати про аксіоми взагалі.
⋮⋮⋮
No.
289699
>>289653>Сучасний підхід це моделювати геометрію всередині ZFC фреймворка (теорії множин), в якому моделюється вся математика.Але при моделюванні всередині ZFC ми ж все одно повинні змоделювати там всередині якісь аксіоми. Тобто аксіоми для геометрії там все одно будуть(тільки вони будуть на рівні моделі всередині ZFC), які для цього беруть аксіоми?
І як це описувати через ZFC? Наприклад, як описувати рівносторонній трикутник через ZFC?
⋮⋮⋮
No.
289702
>>289699>Але при моделюванні всередині ZFC ми ж все одно повинні змоделювати там всередині якісь аксіоми.Так, звісно, будь яка аксіоматична геометрія (хоч Тарського, хоч Гільберта) інтерпретується в ZFC, але не дуже канонічним чином, ми можемо обирати спосіб інтерпретації.
> Наприклад, як описувати рівносторонній трикутник через ZFC?Одна з можливих наступна. Рівносторонній трикутник це невпорядкована трійка пар дійсних чисел {(x1,y1),(x2,y2),(x3,y3)} така що викоується пара рівностей
(x2-x1)^2+(y2-y1)^2 = (x3-x1)^2+(y3-y1)^2
(x2-x1)^2+(y2-y1)^2 = (x3-x2)^2+(y3-y2)^2
⋮⋮⋮
No.
289716
>>289702>Одна з можливих наступна. Рівносторонній трикутник це невпорядкована трійка пар дійсних чисел {(x1,y1),(x2,y2),(x3,y3)} така що викоується пара рівностей>(x2-x1)^2+(y2-y1)^2 = (x3-x1)^2+(y3-y1)^2>(x2-x1)^2+(y2-y1)^2 = (x3-x2)^2+(y3-y2)^2 Треба ще додати умову про неспівпадіння координат всіх трьох точок. І я не бачу тут ніяких множин, це аналітична геометрія
⋮⋮⋮
No.
289721
>>289716Можна хотіти розглядати виродженний трикутник як трикутник. Я не про політично-соціальні назви наукових розділів і дісциплін накшталт "аналітична геометрія", "теорія множин", "теорія чисел", а про математику.
А саме, про формальну теорію першого порядка ZFC і про формальну теорію першого порядка "Акісоми Тарского" і про існування інтерпретації останньої формальної теорії в першу, при якої предикт визначающий проінтерпретованний домен це "U(x) = x є парою дійсних чисел", який виражається в ZFC. Це теорема.
"Аналітична геометрія" не є формальною теорією першого порядку.
⋮⋮⋮
No.
289723
>>289721>А саме, про формальну теорію першого порядка ZFC і про формальну теорію першого порядка "Акісоми Тарского" і про існування інтерпретації останньої формальної теорії в першу, при якої предикт визначающий проінтерпретованний домен це "U(x) = x є парою дійсних чисел", який виражається в ZFC. Це теорема. Тобто є така інтерпретація множин, що через них можна закодувати дійсними числами і якимись рівняннями, зрозуміло.
А якщо через ZFC виразити саму себе, то чи можна засобами ZFC довести, що всі теореми в початковій ZFC це повністью те ж саме, що такі ж самі теореми, записані в цій "ZFC в ZFC"? Тобто що будь-яка теорема, що може бути доведена/спростована/нерозв'язна в ZFC, буде так само доведена/спростована/нерозв'язна в цій "ZFC в ZFC"?
⋮⋮⋮
No.
289724
>А якщо через ZFC виразити саму себе, то чи можна засобами ZFC довести, що всі теореми в початковій ZFC це повністью те ж саме, що такі ж самі теореми, записані в цій "ZFC в ZFC"?
Ні, твердження "З "ZFC доводить 'ZFC доводить (phi)'" випливає "ZFC доводить phi" " еквівалентно тому що ZFC Sigma_1-корректна або протиречива. Останнє не доводеться в ZFC за теоремою Гьоделя. Про це десь було гарно у Жеврабека написано, або десь на МО.
Ще текст про формалізацію.
Хочу додати, що це загальний (сучасний) трюк для формаліста щоб відчути спокій: закодувати це в ZFC (або якась аналогічна сильна теорія, але ZFC то є освітній стандарт), або переконати себе в принциповій можливості такого кодування. Аналітична геометрія з точкі зору формаліста це каркас ZFC-визначень і ZFC-теорем, а вся математика це те що може бути доведено з ZFC. Так само може бути зроблено і з елементарною геометрією: "точка", "конгруентність" і "лежати поміж" перевизначаються в ZFC-термінах і після такого перевизначення аксиоми тарського доводяться з ZFC-аксіом. Плюси такого "конструктивного" підходу на відміну від сінтетичного підходу в тому що ZFC дозволяє ставити питання і використовувати методи інших дисциплин. Наприклад, синтетична геометрія тарського не може навіть сформулювати (не те що довести!) твердження накшталт "для будь якого n, сума внутрішніх кутів n-кутника то є (n-2)pi", тому що в неї нема доступу до натуральних чисел і вона не може сказати "для будь якого n", а у ZFC є.
⋮⋮⋮
No.
289726
>Ні, твердження "З "ZFC доводить 'ZFC доводить (phi)'" випливає "ZFC доводить phi""
"З "ZFC доводить 'ZFC доводить (phi)'" випливає "ZFC доводить phi" для будь якого твердження phi", точніше.
⋮⋮⋮
No.
290250
>>289724>Sigma_1-корректнаА що таке "Sigma_1-корректна", де про це прочитати?
⋮⋮⋮
No.
290253
>>290250Т є Sigma_1-коректною (або Sigma_1-чесною) якщо всі Sigma_1-твердження які доводить Т є істиними. Інакше кажучі, якщо теорія Т доводить що програма (без параметрів) зупеняється, то вона дійсно зупиняється (в реальному світі) Має бути во всіх підручниках логіки в яких доводиться теорема Гьоделя про неповноту. В вікіпедії є визначення
https://en.wikipedia.org/wiki/%CE%A9-consistent_theory мають бути во всіх місця в яких гьодель доводиться я думаю. Тут наприклад
https://www2.karlin.mff.cuni.cz/~krajicek/smorynski.pdf с.844 во внутрішній нумерації.
⋮⋮⋮
No.
290255
Визначення 4.2.4 якщо точніше.
⋮⋮⋮
No.
290259
>>290253>Т є Sigma_1-коректною (або Sigma_1-чесною) якщо всі Sigma_1-твердження які доводить Т є істиними. Інакше кажучі, якщо теорія Т доводить що програма (без параметрів) зупеняється, то вона дійсно зупиняється (в реальному світі) Має бути во всіх підручниках логіки в яких доводиться теорема Гьоделя про неповноту. Я розумію істину в математичному сенсі як можливість довести якесь твердження в якійсь формальній системі. Якщо ми кажемо, що в теорії T твердження х не можна довести, але воно чомусь істинно, то звідки ми маємо впевненість у тому, що воно істинне? Чи може я неправильно розумію значення слова "істина"?
⋮⋮⋮
No.
290262
>>290259> Чи може я неправильно розумію значення слова "істина"?Неправильно думаю, істина це семантичне поняття, воно про виконанность, доведенность це синтаксичне, воно про маніпуляцію зі строками. Багато теорії і глибоких ефектів в мат.лозіці саме про напругу між "істинністю" і "доведенністю", тому ототожнювати їх не треба. Наприклад, предикат доводимості є виразимим в cильних теоріях (кодування гьоделя примітивно-рекурсивних функцій), але предикат істинності не є виразимим (теорема тарского про невиразність істини).
Істина розуміється в мат.лозіці в декількох сенсах, але той сенс про який я кажу, це "істина в стандартній моделі". А саме, для аріфметичних теорій, є стандартна модель: звичайні натуральні числа зі звичайними аріфметичними операціями. Коли я кажу про істинність арифметичного твердження я саме кажу про істинність цього твердження в стандартній моделі. Неформально це значить що арифметичне твердження є... Істиним. Наприклад твердження існує трійка натуральних чисел xyz така що x^2 + y^2 = z^2 є істиним, тому що вона правда існує (3,4,5). Формально його можна визначити в метатеорії, скажимо в ZFC, по рекурсивному визначенню тарського: ZFC вміє говорити про натуральні числа і вона вміє говорити про те що "твердження є істиним в натуральних числах". Тому
>але воно чомусь істинно, то звідки ми маємо впевненість у тому, що воно істинне? Ми можемо бути впевненним в цьому довевши істинность цього твердження в тієї (мета)теорії в якої ми взагалі розмовляємо про істинність і теорії. В неформальній українській мові (наївний платонізм) наприклад, або в ZFC (ZFC-формалізм).
⋮⋮⋮
No.
290271
>>290262> Ми можемо бути впевненним в цьому довевши істинность цього твердження в тієї (мета)теорії в якої ми взагалі розмовляємо про істинність і теорії. В неформальній українській мові (наївний платонізм) наприклад, або в ZFC (ZFC-формалізм). ZFC-формалізм оперує множинами, тобто щоб в ньому формулювати твердження про істинність і теорії, треба якось виразити це через множини, і як бути впевненим в тому, що побудована таким чином (мета)теорія дає вірні відповіді (всі істинні в тій теорії твердження насправді істинні, всі неістинні насправді неістинні)? Ще одну мета-мета теорію робити, яка це доводить, а потім мета-мета-мета... Як взагалі вигадують математичні теорії, які дають правильні відповіді? Для цього ясновидіння якесь потрібно, чи як?
⋮⋮⋮
No.
290272
>>290271>ZFC-формалізм оперує множинами, тобто щоб в ньому формулювати твердження про істинність і теорії, треба якось виразити це через множиниТак, звісно.
> і як бути впевненим в тому, що побудована таким чином (мета)теорія дає вірні відповіді (всі істинні в тій теорії твердження насправді істинні, всі неістинні насправді неістинні)?Ніяк, це теорема Гьоделя. Ми можемо тільки "контролювати ризики". Скажимо люди скептичні відносно множин і нескінченностей можуть залишатись в PA, де є тільки елементарна арифметика в якої кодується дуже багато математики, але не вся. Скажимо в РА формулюється гіпотеза Рімана.
>Як взагалі вигадують математичні теорії, які дають правильні відповіді? Для цього ясновидіння якесь потрібно, чи як?Для переважній більшості математики потрібні дуже слабкі основи, типу PA (аріфметика з індукцією), аксіоми якої дуже складно назвати не істиними, то треба свою шизу особливим чином розігнати. Але в більш екзотичних розділах (теорії множин, логіки), де теорії все ж таки вигадують зазвичай використовують філософські, метаматематичні аргументи щоб вигадувати нові аксіоми. Більшість з цих аргументів підсумована в двух статтях "Believing the Axioms" від Penelope Maddy. Приклад аргумента: аксіома гарна бо з неї виводяться багато інших класичних гарних аксіом, або аксіома гарна бо її не можна заревʼютити форсінгом. Почитай Медді, вона насправді гарно підсумувала відповідь на твоє питання (це філософія в більшості, тому має бути не так складно).
⋮⋮⋮
No.
290330
>>290329Чому це нема?
Ну добре, можна додати умову, що точки знаходяться в квадраті с довжиною сторони a (насправді не важливо, який там розмір того квадрата, вірогідність від цього не змінюється)
⋮⋮⋮
No.
290338
>>290335Це приблизний розрахунок.
Є поправка на те, що замість "<" тої, або іншої координати, буде "≤".
Точка може виходити трішечки за периметр, або бути на лінії трикутника, і при цьому відповідні умови нерівностей дотримуватимуться.
Ще три випадкові точки можуть зовсім не утворити трикутник, а стати точками лінії або повністю співпадати.
Тобто може бути, знову приблизно, не 6,25%, а 3%. І тоді сумарна вірогідність дорівнюватиме не 0,024414062%, а десь 0,0027%.
⋮⋮⋮
No.
290339
>>290338>Це приблизний розрахунок.А точне рішення можна знайти?
>Ще три випадкові точки можуть зовсім не утворити трикутник, а стати точками лінії або повністю співпадати.Вірогідність цього дорівнює 0 тому що координати x y кожної точки - дійсні числа.
⋮⋮⋮
No.
290340
>>290339Є подібна математична проблема:
"З внутрішньої сторони описаного кола рівностороннього трикутника навмання береться точка. Яка ймовірність того, що вона лежить усередині описаного [навколо трикутника] кола, але поза вписаним всередину трикутника колом?"
Відповідь: (pi
R^2 - pi r^2) / pi * R^2 = (4a^2 / 3 - a^2 / 3) / (4a^2 / 3) =
3/4https://www.doubtnut.com/question-answer/a-point-is-taken-at-random-from-inside-of-the-circumcircle-of-an-equilateral-triangle-the-probabilit-630436906"Щоб дізнатися, чи є точка (x, y) на графіку прямої, ми підставляємо значення та дивимося, чи ми отримуємо істинне твердження, наприклад 10 = 10."
Тобто якщо точка A (2,3) знаходиться на прямій DT, стороні трикутника, то:
y {D_y...T_y} = х {D_x...T_x} + b,
І тут допустиме співпадіння значення одної з координат точки A. Але не двох одночасно.
3 =/= 2 + b => b =/= 1.
Якщо точка A виходить за межі периметру DTH, то:
"Найпростішим способом визначити, чи лежить точка всередині трикутника, є перевірка кількості точок в опуклій оболонці вершин трикутника, прилеглих до даної точки. Якщо оболонка має три точки, то ця точка лежить усередині трикутника; якщо чотири, то лежить поза трикутником."
Також є різноманітні алгоритмічні способи тестування:
http://totologic.blogspot.com/2014/01/accurate-point-in-triangle-test.htmlCheck if a point lies inside a triangle
⋮⋮⋮
No.
290353
>>290330>Чому це нема?Якщо би було, то яка мала би бути ймовірність того що випадкове число с таким розподілом опиниться на інтервалі (-n,n)? Якщо відповідь буде що 0, то взявши границі можна побачити що ймовірність достовірної події 0, що є протеріччя. Багато ймовірностних парадоксів побудовано на цьому, парадокс про два конверти скажимо.
Для квадрата зі стороною 1: ймовірність це (площа випадкового трикутника)^3. Площа випадкового трикутника це (1/2)
int_{[0,1]^6} |(x2-x1)(y3-y1)-(x3-x1)*(y2-y1)| dx1...dy3 = 11/144. Тобто відповідь (11/144)^3 (якщо все правильно порахував).
⋮⋮⋮
No.
290365
>>290353>Площа випадкового трикутника це (1/2)int_{[0,1]^6} |(x2-x1)(y3-y1)-(x3-x1)*(y2-y1)| dx1...dy3 = 11/144Я не дуже розумію цей запис. Ти інтегруєш розподіл імовірностей для площі випадкового трикутника? А як накладати одну вірогідність на іншу, якщо числа перемножують? От наприклад, якщр є два дійсні числа, кожне від 0 до 1 з рівномірним розподілом, то якою формулою описується їх добуток?
⋮⋮⋮
No.
290366
Як уявити 4й вимір?
⋮⋮⋮
No.
290367
>>290365Це мат.сподівання випадкової величини "площа трекутника з вершинами (x1,y1),(x2,y2),(x3,y3)". Всередині інтеграла записана формула (Гаусса) площини трекутника по координатам: половина модуля псевдоскалярного добутка, щоб знайти матсподівання треба функцію площина(x1,...,y6) помножити на (плотність) імвоірности сценарія (x1,...,y6) (яка 1, для рівномірного розподіла) і проінтегрувати по всім можливим сценаріям. Формально це (майже) визначення, неформально це має сенс: щоб знайти середнє випадкової величини ти тестуєш кожний сценарій і результат теста домножуєш на ймвоірність цього сценарія і береш суму.
Розподіл добутка на вікіпедії є наприклад
https://en.wikipedia.org/wiki/Distribution_of_the_product_of_two_random_variables але це питання іррелевантно до цієї задачі.
⋮⋮⋮
No.
290374
>>290366Просто додаєш ще одну площину до трьохвимірного простору і ось усе. Важко осягнути, втім..
Як по трьом плоским проекціям уявляють щось об'ємне, так по 6(?) об'ємним проекціям можна уявити 4D об'єкт.
Але скоріше за все тобі просто заболить голова, як боліла мені коли на нарисній геометрії пропонували уявити у русі перетин торової поверхні з конусом.
⋮⋮⋮
No.
290381
>>290367> Це мат.сподівання випадкової величини "площа трекутника з вершинами (x1,y1),(x2,y2),(x3,y3)". Всередині інтеграла записана формула (Гаусса) площини трекутника по координатам: половина модуля псевдоскалярного добутка, щоб знайти матсподівання треба функцію площина(x1,...,y6) помножити на (плотність) імвоірности сценарія (x1,...,y6) (яка 1, для рівномірного розподіла) і проінтегрувати по всім можливим сценаріям.Тобто це 9 разів(по x1 x2 x3 y1 y2 y3) інтеграл від формули для знаходження площі трикутника? Ми ніби перебираємо всі можливі варіанти координат точок, рахуємо площину для кожної і сумуємо? Але потім же треба поділити цю суму на кількість трикутників, для яких ми таким перебором порахували площину, а цієї кількості нема... щось я не розумію
⋮⋮⋮
No.
290390
>>290381>Тобто це 9 разів(по x1 x2 x3 y1 y2 y3)Наплутав, 6 разів (x1 y1 x2 y2 x3 y3)
⋮⋮⋮
No.
290391
>>290381Вибач, я зараз зрозумів що попереднє моє рішення є неправильним. Я відповів на задачу "обираємо 3 випадкових трикутника і 3 точки, яка ймовірність того що 1 точка в 1 трикутнику, 2 точка в 2, а третя точка в 3"? В цьому випадку це (11/144)^3, але відповідь на твою задачу це
(1/8) integral_{[0,1]^6} |(x2-x1)(y3-y1)(x3-x1)(y2-y1))|^3 dx1dx2x3dy1dy2dy3 = 11/4032
відповідь тепер сходиться з моєю (криво написанною) симуляцією
https://pastebin.com/7AZeg48p
⋮⋮⋮
No.
290394
>>290381Відповідаючи на твоє запитання. Зафіксуємо трикутник Delta. Яка ймовірність того що три навмання обрані точки попадуть в цей трикутник? Це Площа(Delta)^3. Тепер уявимо що трикутників скінченна кількість Delta_1,...,Delta_n і трикутник p_i випадає з ймовірністтю Delta_i. І ми спочатку навмання обираємо навмання трикутник а потім тестуємо три точкі, яка тоді відповідь на задачу буде?
Буде Sum_{i=1}^n p_i Площа(Delta_i)^3. Тепер неперервизація задачі: нам потрібно замінити суму на інтеграл, дискретний інтервал {1,...,n} на множину всіх трикутників: [0,1]^6 а ймовірність на плотність ймовірності 1.
⋮⋮⋮
No.
290403
(1/8) integral_{[0,1]^6} |(x2-x1)(y3-y1)-(x3-x1)(y2-y1))|^3 dx1dx2x3dy1dy2dy3 = 11/4032
точніше
⋮⋮⋮
No.
290452
ВИБАЧТЕ, безоси, в симуляції і в підрахунку інтеграла була помилка. Ось виправлена симуляція
https://pastebin.com/AcqWHRLu і виправлений підрахунок
https://pastebin.com/WnfVfNw9 (Wolfram Mathematica) відповідь: 137/72000. Зараз би в задачі для 2го курса 2 рази помилитись, жах і сором.
⋮⋮⋮
No.
290536
>>290452А як ти ту формулу інтеграла виводив (останню, яка на твій погляд правильна)?
Що до симуляції: можна взяти випадковий трикутник, всі точки якого в межах від 0 до 1 по x, y, порахувати те, яка в нього площа по відношенню до площі квадрата зі стороною 1 (тобто ділимо на 1 що нічого не міняє, це буде вірогідність потрапляння в той трикутник однієї точки, як метод Монте-Карло, тільки навпаки), потім ведення у третій ступінь, це буде вірогідність трьох влучань.
Ось така симуляція виходить
https://pastebin.com/tKjCnwTN - відповідь 0.001903 - це схоже на 137/72000
тобто треба 6 разів для x1 x2 x3 y1 y2 y3 інтегрувати вираз
((x2-x1)
(y3-y1)-(x3-x1)(y2-y1))/2)^3
так?
А як розуміти те, що в
https://pastebin.com/WnfVfNw9 ? Що таке s, t, g? Звідки 6/8?
⋮⋮⋮
No.
290537
>>290536> ((x2-x1)(y3-y1)-(x3-x1)(y2-y1))/2)^3((x2-x1)(y3-y1)-(x3-x1)(y2-y1))/2) по модулю звісно
⋮⋮⋮
No.
290573
>>290537>тобто треба 6 разів для x1 x2 x3 y1 y2 y3 інтегрувати вираз (|(x2-x1)(y3-y1)-(x3-x1)(y2-y1)|/2)^3 так?Так.
>А як розуміти те, що в https://pastebin.com/WnfVfNw9 ? Що таке s, t, g? Звідки 6/8?Так, ідея наступна. Нехай (a1,a2),(b1,b2),(c1,c2) це трійка випадково обраних точок. Будемо разглядати тільки випадки a2<b2<c2 і домножимо відповідь на 6 щоб врахувати всі інші випадки (6 можливих перестановок з трьох елементів). Тепер, нехай точка (s,b2) це перетин прямої y=b2 з відрізком (a1,a2),(c1,c2). Тоді площа трикутника це (с2-a2)|s-b1|/2, а нас цікавить інтеграл від ((с2-a2)|s-b1|/2)^3 по всім можливим параметрам. А саме {b1,0,1}, {b2,a2,c2},{c1,0,1},{a1,0,1},{c2,a2,1},{a2,0,1}.
В першій строчці я раскриваю модуль |s-b1|^3.
В другій і (частково) третій: інтегрування {b2,a2,c2}: s в нас залежіть від b2. А саме:
s = (1-t) a1 + t c1 для деякого t
b2 = (1-t) a2 + t c2 для того ж самого t
тому d b2 = (c2-a2) dt, звідси множник (c2-a2) в третій строчці. Інтегрування {b2,a2,c2} після заміни стає інтегруванням {t,0,1}.
Во всіх інших інтегруваннях нема що коментувати.
Таким чином, я інтегрую тількі многочлени, що простіше ніж якийсь вираз з модулем, який Вольфрам Математика не може порахувати. До речі, це приклад коли знаючи природу задачі людина (я) може порахувати якийсь інтеграл, а саме (|(x2-x1)(y3-y1)-(x3-x1)(y2-y1)|/2)^3 dx1...dy3, а комп не може.
⋮⋮⋮
No.
290615
>>290573> До речі, це приклад коли знаючи природу задачі людина (я) може порахувати якийсь інтеграл, а саме (|(x2-x1)(y3-y1)-(x3-x1)(y2-y1)|/2)^3 dx1...dy3, а комп не може.Може є якісь програми, які беруть інтеграли краще? Чи існує алгоритм, яким можна було б завжди проінтегрувати, якщо це принципово можливо?
⋮⋮⋮
No.
290617
>>290615>Може є якісь програми, які беруть інтеграли краще?Може!
> Чи існує алгоритм, яким можна було б завжди проінтегрувати, якщо це принципово можливо?Існує, це алгорітм Ріша який завжди бере
невизначений інтеграл в елементарних коли такий існує, або каже це неможливо (за модулем проблеми констант, але це не важливо). Наскільки я знаю, він досі не імплементований повністю. Для визначенних інтегралів такого алгорітма не відомо навіть близько, бо існування такого алгорітма би вирішило (або би підійшло дуже близько до вирішення) гіпотезу Гротендика о перійодах.
⋮⋮⋮
No.
290618
>це алгорітм Ріша який завжди бере невизначений інтеграл в елементарних коли такий існує
І тільки для однієї змінної, для декількох змінних що таке "невизначенний інтеграл" трохи меньш зрозуміло. "Пучок рішень диференціального рівняння" найбільш природнє узагальнення, але (мені) не дуже зрозуміло як таке в компі кодувати навіть. Яким чином модифікувати алгорітм Ріша щоб він з таким працював зрозуміло ще меньш. Але тут я не експерт, може і є стандартні рішення. Швидким пошуком не знайшов.
⋮⋮⋮
No.
290645
>>290573> До речі, це приклад коли знаючи природу задачі людина (я) може порахувати якийсь інтеграл, а саме (|(x2-x1)(y3-y1)-(x3-x1)(y2-y1)|/2)^3 dx1...dy3, а комп не може. А чи можна через CAS якимось чином довести через якісь символьні трансформації, що integral (|(x2-x1)(y3-y1)-(x3-x1)(y2-y1)|/2)^3 dx1...dy3 рахує те ж саме, що рахує
https://pastebin.com/WnfVfNw9 ? Яка CAS краще всього бере інтеграли? Тобі в роботі ці CAS допомогають якось?
⋮⋮⋮
No.
290647
>А чи можна через CAS якимось чином довести через якісь символьні трансформації, що integral (|(x2-x1)(y3-y1)-(x3-x1)(y2-y1)|/2)^3 dx1...dy3 рахує те ж саме, що рахує https://pastebin.com/WnfVfNw9 ?Не знаю, думаю ні, я не чув про таке принайні. Зазвичай те як CAS щось рахує то її внутрішня справа. Я не чув про якийсь функції які відповідали би на питання накшталт "чи можна дві символьні репрезентації звести одна до одної заміною змінних", а тут ще гірше, бо в мойому пастбіні воно навіть не в одну строчку накшталт Integral[F,...] написано, а в чотири.
>Яка CAS краще всього бере інтеграли? Не знаю. За відчуттями якщо одновимірний невизначений то Axiom найкращий, бо там більше гілок алгорітму Ріша реялізовано. Якщо многовимірні визначені (як цей) то Mathematica накйркаща, бо пару разів воно мені доволі складні речі рахувало.
> Тобі в роботі ці CAS допомогають якось?Так, але я більше використовую Macaulay2 бо рахувати треба не інтеграли а всяку лінійно-алгебричну фігню, конуси, політопи, базіси грьобнера ідеялів і всяке таке. Нещодавно CYTools теж корисно виглядає.
⋮⋮⋮
No.
290649
249
⋮⋮⋮
No.
307360
>>282334Абсолютно з тобою незгоден. А 10 і 11 клас це взагалі повторення того, що впихували прискорено у 9му.
Вважаю, що за ці два останніх роки можна було розповісти ще більше.
Казати, що частина матеріалу непотрібна.. якщо це база, причому екстракт з бази урізаний на стільки, щоб стати загальнодоступним. Чєл ти.. Універу це теж стосується, той бакалавр, що у нас, його можна до 2-3 років скоротити спокійно, взагалі без вагань. Магістратура так це взагалі нікчемна параша, так, для галочки. А ти кажеш, що тобі щось ще й непотрібне 🤯
⋮⋮⋮
No.
307362
>>307360Я вчився давно але в 10 і 11 точно був новий матеріал, стереометрія щонайменш. Взагалі програма як вона прописана є доволі потужною. Люди мають вміти рахувати границі, досліджувати функцію за допомогою похідної, рахувати обʼєми і площу тіл обертання: тобто знати матеріал першого семестру першого курсу по факту. В інформатиці взагалі сміх, здається що писав якийсь айтішнік: люди мають знати алгоритм флоуда-уоршелла, робити запити на моаві SQL, створювати веб-сайти, проєектувати інтерфейси і документацію програмних продуктів. Це не жарт ось
https://mon.gov.ua/ua/osvita/zagalna-serednya-osvita/navchalni-programi/navchalni-programi-dlya-10-11-klasiv . Але не все що написано в вордівському файлі на сайті мін.освіти автоматично втілюється в життя. Що треба зробити зараз для підвищення рівня освіти: повернутися до ЗНО якнайшвидше, бо НМТ це сміх.
⋮⋮⋮
No.
307372
>>307360>>307362Худ літ малювання і тому подібне що найменш мають не оцінюватись, а по нормальному виключені з обовязкових, щоб викладачі з цих придметів пішли нахуй на смітники збирати пляшки і картонки.
⋮⋮⋮
No.
307376
>>307372На мою думку література - не має, особливо враховуючи культурний тиск який на нас роблять. Малювання це ж взагалі до 7 класу чи щось таке, тобто воно більше як розвага для дітей і є зараз. Що можна робити: це давати свободу самому обирати і будувати свою програму вчителям в крупних містах типу Києва, Одеси, Харкова (можливо з якимось додатковими рестрікшинами на вивчення російської мови і літератури). В селах, селищах і смт такої свободи звісно давати не треба, бо тоді вони читать розучаться.
⋮⋮⋮
No.
307377
>>307376Як ти взагалі пропонуєш оцінювати читання якоїсь нудної нудяти поки однолітки на вулиці гуляють і радіють життю?
Це ж психологічно моральне насильство.
⋮⋮⋮
No.
307378
>>307377Обовʼязкова школа так чи інакше буде присутня в житті підлітка він не зможе нонстоп гуляти, як і його однолітки, питання тільки в тому що включати в шкільну программу а що ні. Якщо не секрет: ти школьник?
⋮⋮⋮
No.
307379
>>307378> ти школьник?Їбе?
> Обовʼязкова школаА якщо обовязково буде лгбт пропаганду в школі робити з наступом західних тенденцій?
⋮⋮⋮
No.
307380
>>307379Ну якщо запитав то їбе, бо якщо би не їбало то й не питав би, егеж?
>А якщо обовязково буде лгбт пропаганду в школі робити з наступом західних тенденцій?То так і буде, не зрозумів в чому питання.
⋮⋮⋮
No.
307381
>>307380> Ну якщо запитав то їбе, бо якщо би не їбало то й не питав би, егеж?Поїбе і перестане.
> То так і буде, не зрозумів в чому питання. А якщо бдсм практики з вчителями зроблять обовязковими? (ніби зараз так не є, просто не офіційно)
⋮⋮⋮
No.
307382
>>307381Міг би сказати що секрет, але за твоєю реакцією і стилем спілкування очевидно що ти шкіла. Радий що ти на кропиві.
>А якщоА якщо в дупі пальцем колупатись то смердіти буде, козаче. А якщо думку висловити хочеш, то будуєш стверджувальне речення, замість риторичного питання (бо так тільки фємки-лгбт-сжв-трансухи-ліваки роблять).
⋮⋮⋮
No.
307383
>>307382> А якщо в дупі пальцем колупатись то смердіти будеДванок колупаєшся в дупі? Сильно смердить?
> замість риторичного питання Діалектика вона взагалі така - безпосщадний Окама чи як там його.
⋮⋮⋮
No.
354438
>>274298Поясни за корінь з мінус одиниці, раніше бачив цю хуйню, якаось звязана з коренем з двох, але ось подивився фільм Платформа 2, то там жирний тіп піздів що це уявне число і якусь хуйню що значить вся математика йде нахуй тому, в що мало віриться.
Короче давай, підписався пояснювати то пояснюй.
⋮⋮⋮
No.
354440
>>354438Коли ми кажемо, що "√4 = ±2", ми маємо на увазі, що або "2 × 2 = 4", або "-2 × -2 = 4".
Тоді як "√-4 × √-4 = -4", проте -4 × -4 = 16, дійсно.
У складних рівняннях інколи не потрібно витягувати число з під кореня, де воно перебуває "у відносній невизначеності".
І коли ми використовуємо "√-4" для розрахунків, ми маємо пам'ятати, що воно має значення лише коли інше негативне число під коренем присутнє, аби скласти пару і компенсувати "неправильність".
Якщо цікаво, ось інформація про практичне застосування комплексних чисел:
"Управління повітряним рухом використовує радар (RAdio Detection And Ranging / РАДіо визначення та Ранжування) для виявлення рухомих літаків.
Коли ми думаємо про радар, впевнений, багато хто уявлятиме обертову зелену голку, яка обертається навколо чорного круглого диска, як показано нижче.
Центр управління має обертове джерело, яке сканує на 360º кожні 2-3 секунди.
Джерело надсилає радіосигнал із частотою, вищою за ті, які використовуються для радіо, чи телевізійних передач, щоб уникнути перешкод, і має приймач, який виявляє відлуння від будь-якого об’єкта на своєму шляху.
Ви можете думати про це як про кидання м’яча в темну кімнату і спостерігати, чи повернеться він до вас, чи ні.
Якщо це так, то щось має бути перед вами, і чим довше потрібно, щоб повернутися, тим далі має бути об’єкт.
Замість того, щоб використовувати щось маленьке, як м’яч, радіопромінь має форму віяла: вузький у горизонтальному напрямку та широкий у вертикальному напрямку, щоб охоплювати велику площу.
Цей метод відомий як первинний радар.
Однак сьогодні він використовується рідко, тому що в небі зараз занадто багато літаків.
Тож замість цього ми використовуємо вторинний радар, де кодована послідовність імпульсів надсилається до літака, а транспондер у літаку генерує закодовану відповідь.
Але яке відношення це має до комплексних чисел?
Ну, уявіть, яку кількість даних повинен обробляти комп’ютер, якщо центр управління може охопити все коло за 2-3 секунди.
Крім того, радар сканує літаки, які можуть рухатися зі швидкістю до 900 км/год, що ускладнює обчислення.
Використання уявних чисел дозволяє комп’ютерам обчислювати набагато швидше.
Ті самі обчислення можна зробити з дійсними числами, але літак перемістився б кудись в іншому місці до того моменту, як обчислення буде виконано!
Дані, які отримують центри управління повітряним рухом, часто містять багато шуму, і іноді радару важко вловити сигнал від літака, подібно до того, як важко почути, як хтось говорить поруч із вами в гучній кімнаті.
Але є способи, за допомогою яких ми можемо видалити фоновий шум, щоб зробити сигнал із літака чіткішим. Частота фонового шуму зазвичай відрізняється від частоти плоского сигналу, і ми можемо розрізнити ці частоти за допомогою так званого перетворення Фур’є.
Я не буду вдаватися в математику, тому що це стає досить складним, але теорія перетворень Фур’є в основному говорить, що випадковий візерунок може бути створений косинусними та синусоїдальними хвилями різних амплітуд і частот, і це говорить вам, що ці різні хвилі.
[
https://www.wikiwand.com/uk/articles/Обернена_теорема_Фур'є]
Як ви бачите у формулі, для використання перетворення Фур’є потрібні комплексні числа.
Отже, питання полягає в тому, яке відношення мають функції косинус і синус до комплексних чисел? Ось тут і з’являється формула Ейлера:
[
https://www.wikiwand.com/uk/articles/Формула_Ейлера]
Якщо це все для вас нове, ви, безсумнівно, думаєте, який сенс це все має? Звідки це взялося? Відповідь міститься на діаграмі Арганда.
[
https://www.wikiwand.com/uk/articles/Комплексна_площина]
Якщо ми подивимося на діаграму одиничного кола нижче, то побачимо, що ми можемо представити дійсну та уявну частини комплексного числа за допомогою cos і sin.
Це схоже на те, як ми можемо виразити x = cosθ і y = sinθ у нашій звичайній координатній площині, за винятком того, що x і y є дійсними та уявними розмірами відповідно на діаграмі Арганда.
Ми називаємо e iθ полярною формою, оскільки тут ми представляємо комплексне число його кутом і радіусом (у цьому випадку радіус = 1).
Подібним чином ми називаємо cosθ + i sinθ, який має форму a + bi, прямокутною формою."
"Уявні числа також можуть допомогти нам краще інтерпретувати хвилі.
Думаючи про хвилі, більшість людей уявляють собі періодичні рухи вгору та вниз по сторінці.
Однак ми також можемо думати про хвилю як про зміну координати x або y під час руху по колу.
Фактично, звідси походять функції cos(x) і sin(x).
Але є проблема–схожа на вигляд хвиля насправді може представляти два типи руху.
Ми називаємо їх позитивними та негативними частотами.
Це можна пояснити за допомогою концепції візуалізації хвиль у вигляді кола, як зазначено вище.
Як ми зазвичай думаємо про хвилі, ми беремо лише один вимір (x або y) кола.
Що, якщо ми хочемо включити обидва виміри? Це те, що дозволяє нам робити уявний вимір.
Якщо ми приймемо x і y як реальні та уявні розміри, ми створимо спіраль.
Хвиля в основному є проєкцією спіралі лише в реальний вимір.
За допомогою спіралі ми можемо побачити, чи має хвиля позитивну чи негативну частоту: хвиля виглядає однаково в реальному вимірі, але має протилежні напрямки в додатковому уявному вимірі."
"Коли диспетчерська вежа отримує ці хвилі, вона перетворює уявні частини хвиль назад у реальний вимір.
Спосіб, яким ми виконуємо це перетворення, є додавання хвилі до її спряженого.
Якщо ми маємо комплексне значення a + bi, додавання спряженого a – bi дає нам (a + bi) + (a – bi) = 2a, подвоєний дійсний компонент.
З врахуванням комплексного числа та його спряженим, ми також можемо називати їх позитивною (a + bi) і негативною частотами (a – bi).
Якщо ви поглянете на графіки вище, ви побачите, що хвилі в уявних вимірах протилежні одна одній через +b і -b у комплексному числі та його спряженій формі.
Фактично, радіохвиля кодує ці 2 частини інформації разом, так що інформація може бути збережена.
Дивно, як уявні числа, які важко уявити, можуть мати таке важливе застосування.
«Уявні» числа зовсім не є уявними, інакше авіаперельоти були б неможливими!"
Комплексні числа, управління повітряним рухом і РАДАР (19.10.2021):
https://tomrocksmaths.com/2021/10/19/air-traffic-control-and-radar/